Download as docx, pdf, or txt
Download as docx, pdf, or txt
You are on page 1of 71

Endocrine Surgery MCQ

1. When progressive enlargement of a multinodular goiter causes


symptomatic tracheal compression, the preferred management in otherwise
good-risk patients is:
A. Iodine treatment.
B. Thyroid hormone treatment.
C. Surgical resection of the abnormal thyroid.
D. Radioactive iodine treatment.
Answer: C

DISCUSSION: When a multinodular goiter enlarges enough to cause


symptoms of tracheal compression, surgical treatment is usually required if
the patient is considered a reasonable operative risk. Medical treatment may
be effective in preventing the initial growth of the goiter but is unlikely to
cause enough regression to relieve symptoms. Radioactive iodine can
occasionally be used to cause some regression in patients who are poor
anesthesia risks, but this is a temporizing treatment rather than a definitive
one.

2. The most precise diagnostic screening procedure for differentiating benign


thyroid nodules from malignant ones is:
A. Thyroid ultrasonography.
B. Thyroid scintiscan.
C. Fine-needle-aspiration biopsy (FNAB).
D. Thyroid hormone suppression.
Answer: C
DISCUSSION: Analysis of multiple series in which patients with thyroid
nodules have undergone FNAB has demonstrated a false-negative rate of

2.4% and a false-positive rate of 3.3%. Sensitivity for this method is 92%;
specificity 74%. This surpasses the other methods for accurate selection of
patients who require surgical resection.

3. The preferred operation for initial management of a thyroid nodule that is


considered suspicious for malignancy by FNAB is:
A. Excision.
B. Partial lobectomy.
C. Total lobectomy and isthmusectomy.
D. Total thyroidectomy.
Answer: C
DISCUSSION: There is a consensus that the initial minimum operation for a
nodule suspected to be malignant is total lobectomy and isthmusectomy.
Partial lobectomy or excision of the nodule is associated with a higher risk of
local recurrence if the nodule proves to be malignant. Reoperation on the side
of a partial lobectomy can be technically difficult and associated with a
higher risk of recurrent nerve injury. Ordinarily, total thyroidectomy is not
performed until a conclusive diagnosis of malignancy is established.

4. Advantages of total thyroidectomy for management of papillary


carcinomas of the thyroid larger than 1.5 cm. include:
A. Possibility of using radioactive iodine postoperatively to identify and treat
metastases.
B. The ability to use thyroglobulin levels as a marker for recurrence.
C. Lower overall recurrence rate.
D. Lower risk of hypoparathyroidism.
Answer: ABC
DISCUSSION: Following total thyroidectomy iodine 131 can be used more

efficiently because of the absence of normal thyroid tissue, which has greater
affinity for iodine than papillary carcinoma tissue. When all normal thyroid
tissue is removed, serum thyroglobulin, which is produced by normal and
malignant thyroid tissue, becomes a more effective marker for recurrence.
The overall recurrence rate is lower for patients undergoing total
thyroidectomy, but the risk of hypoparathyroidism is higher for patients who
have total thyroidectomy instead of unilateral lobectomy.

5. Which of the following statements about follicular carcinoma is/are true?


A. It presents at a later age than papillary carcinoma.
B. It disseminates via hematogenous routes.
C. It is the most common type of well-differentiated thyroid carcinoma.
D. Extensive angioinvasion portends a poor prognosis.
E. Follicular carcinomas are frequently multicentric.
Answer: ABD
DISCUSSION: Follicular carcinoma is more common in older patients (peak
incidence in the fifth decade). The tumor has a marked propensity for
vascular invasion and spreads hematogenously to bone, lung, liver, and
central nervous system sites. Local nodal metastases are less common than in
papillary carcinoma. Extensive angioinvasion indicates a less favorable
prognosis. Papillary carcinoma is the most common type of welldifferentiated thyroid carcinomas. Follicular carcinomas are rarely
multicentric.

6. A familial form of medullary thyroid carcinoma (MTC) should be


suspected whenever:
A. The tumor is multifocal.
B. The tumor is bilateral (foci of tumor are present in both thyroid lobes).
C. Pathologic examination of the resected thyroid gland reveals the presence

of C-cell hyperplasia in areas of the gland adjacent to foci of MTC.


D. All of the above.
Answer: D
DISCUSSION: Sporadic MTC is unilateral in at least 80% of cases.
However, in patients with MTC occurring as a component of the multiple
endocrine neoplasia (MEN) type 2A or type 2B syndromes, the tumor is
virtually always multifocal and bilateral. Typically, in this setting the MTC
appears as multiple whitish-tan tumor nodules in the middle and upper thirds
of each thyroid lobe.
A diffuse premalignant proliferation of the C cells of the thyroid is thought to
precede the development of MTC in patients with familial MTC. This
proliferation, known as C-cell hyperplasia (CCH), consists of parafollicular
clusters of increased numbers of C cells. The finding of CCH in areas of the
thyroid adjacent to gross foci of MTC is strong evidence for familial MTC.

7. All of the following are components of the MEN type 2B syndrome


except:
A. Multiple neuromas on the lips, tongue, and oral mucosa.
B. Hyperparathyroidism.
C. MTC.
D. Pheochromocytoma.
Answer: B
DISCUSSION: MTC and pheochromocytoma occur in both MEN 2A and
MEN 2B syndromes. Patients with MEN 2A may also develop hyperplasia of
the parathyroid glands. Although some investigators have reported equivocal
histologic abnormalities in the parathyroid glands of patients with MEN 2B,
hyperparathyroidism is not a component of this syndrome.
In contrast to patients with MEN 2A, those with MEN 2B have a
characteristic phenotype, including a tall, thin marfanoid habitus. Patients

with MEN 2B also develop multiple neuromas on the lips, tongue, and oral
mucosa, creating the appearance of thick lips.

8. MEN 2A and MEN 2B syndromes are associated with germline mutations


in:
A. The p53 tumor suppressor gene.
B. The H-ras gene.
C. The N-myc gene.
D. The RET proto-oncogene.
Answer: D
DISCUSSION: Germline mutations in the RET protooncogene, a receptor
tyrosine kinase that maps to chromosome 10, are associated with MEN 2A
and MEN 2B syndromes. Homozygous loss of the tumor suppressor gene p53
is associated with the Li-Fraumeni syndrome, and mutations of p53 are
present in a variety of human neoplasms. Point mutations in the H-ras gene
are associated with carcinoma of the colon, lung, and pancreas. Amplification
of the N-myc gene, when present in neuroblastoma, suggests a poorer
prognosis.

9. Which of the following are true concerning islet cell neoplasms of the
pancreas in patients with MEN type 1?
A. Islet cell neoplasms in patients with MEN 1 are characteristically
multicentric.
B. The most common islet cell neoplasm in patients with MEN 1 is
gastrinoma.
C. Islet cell neoplasms in patients with MEN 1 may be malignant.
D. All of the above.
Answer: D

DISCUSSION: The pathologic change in the pancreas of patients with MEN


1 is typically multicentric. Diffuse hyperplasia of islet cells and
microadenoma formation are often identified in areas of the gland distant
from grossly evident tumor. Tumors are commonly multifocal. Islet cell
neoplasms of the pancreas occur in 30% to 80% of patients with MEN 1. The
most common islet cell neoplasm in these patients is gastrinoma.
Gastrinomas associated with MEN 1 probably account for 20% to 50% of all
cases of the Zollinger-Ellison syndrome. The second most common islet cell
tumor is insulinoma. Other pancreatic islet cell neoplasms, such as
glucagonoma, somatostatinoma, or vasoactive intestinal polypeptide
neoplasm (VIPoma), are rarely associated with MEN 1. Approximately 10%
of insulinomas and approximately 15% or more of gastrinomas in patients
with MEN 1 are malignant.

10. Which of the following statements about the differential diagnosis of


hypercalcemia is/are correct.
A. Malignant tumors typically cause hypercalcemia by ectopic production of
parathyroid hormone (PTH).
B. The diagnosis of primary hyperparathyroidism is supported by these serum
levels: calcium, 10.8 mg. per dl.; chloride, 104 mmol. per liter; bicarbonate
21 mmol. per liter; phosphorus, 2.4 mg. per dl.; elevated parathyroid
hormone.
C. Familial hypocalciuric hypercalcemia is distinguished from primary
hyperparathyroidism by parathyroid imaging.
D. Although serum albumin binds calcium, the measured total calcium value
is usually unaffected in patients with severe hypoproteinemia.
E. Thiazide diuretics are a good treatment for hypercalcemia and can be
given to patients with apparent hypercalcemia of malignancy.
Answer: B
DISCUSSION: Malignant tumors rarely secrete PTH itself; they can secrete

PTHrP or cytokine activators of osteoclast activity. The diagnosis of primary


hyperparathyroidism is supported by hypercalcemia with mild
hyperchloremic metabolic acidosis and a chloride-phosphate ratio greater
than 33 or a modified chloride (mmol. per liter/mg. per dl.) phosphate ratio
greater than 500. Familial hypocalciuric hypercalcemia is distinguished from
primary hyperparathyroidism by a low urine calcium. Serum calcium changes
approximately 0.8 mg. per dl. for every 1 gm. per dl. change in serum
albumin. Thiazide diuretics can cause hypercalcemia and should not be given
to patients who are hypercalcemic.

11. Indications for operation in a patient with previously asymptomatic


hyperparathyroidism include:
A. Age older than 60.
B. Nephrolithiasis.
C. A substantial decline in renal function.
D. A substantial decline in bone mass.
E. Depression and fatigue.
Answer: BCDE
DISCUSSION: Age younger than 50 is considered an indication for
operation.

12. The parathyroid glands:


A. Develop from the second and third pharyngeal pouches, along with the
palatine tonsil and the thymus.
B. Migrate caudally in the neck in normal development but can be found
anywhere from the pharyngeal mucosa to the deep mediastinum.
C. Secrete PTH and calcitonin to manage calcium homeostasis.
D. Usually number four, but frequently number only two or three.
E. Contain enzymes that catalyze the conversion of 25(OH) vitamin D 3 to

1,25(OH) 2 vitamin D 3.
Answer: B
DISCUSSION: The parathyroid glands develop from the third and fourth
pharyngeal pouches, along with the thymus and the thyroid ultimobronchial
body; there are four glands in the vast majority of persons. Calcitonin is
secreted by the C cells of the thyroid. Vitamin D 3 hydroxylation occurs in
the kidney.

13. Hyperparathyroidism can affect which of the following organs and body
systems?
A. Gastrointestinal tract.
B. Kidneys.
C. Skeleton.
D. Neuromuscular system.
E. Cardiovascular system.
Answer: ABCDE
DISCUSSION: Gastrointestinal involvement includes pancreatitis and peptic
ulcer disease. The kidneys can be affected by nephrocalcinosis or
nephrolithiasis. The bones can be severely affected by bone resorption.
Neuromuscular complaints can include weakness and fatigue. Hypertension
is present in as many as 70% of people with hyperparathyroidism and may be
related to the renal disease.

14. Secondary hyperparathyroidism:


A. Is a metabolic disease in which the primary abnormality is decreased
glomerular filtration rate.
B. Is best treated initially by subtotal parathyroidectomy.
C. Is caused by increased production of 1,25(OH) 2 vitamin D 3, causing

increasing intestinal calcium absorption and hypercalcemia.


D. Can have severe effects on bones exacerbated by aluminum contained in
phosphate binders and dialysate water.
E. Is best treated initially by total parathyroidectomy with
autotransplantation.
Answer: AD
DISCUSSION: Secondary hyperparathyroidism is caused by renal disease,
including decreased glomerular filtration rate and decreased 1,25(OH) 2
vitamin D 3 production. It is best treated by medical management restricting
dietary phosphate, administering vitamin D and calcium with phosphatebinding gels, and limiting aluminum exposure, which can exacerbate bone
disease.

15. Hypoparathyroidism:
A. Is most commonly encountered as a postviral syndrome.
B. Can be associated with marked hypocalcemia after parathyroidectomy in
patients with bone disease.
C. Can cause anxiety, depression, or confusion.
D. Can cause physical signs such as Chvostek's and Trousseau's signs.
E. Is treatable acutely with intravenous calcium salts and chronically with
oral calcium and vitamin D.
Answer: BCDE
DISCUSSION: Hypoparathyroidism is most commonly encountered after
thyroid surgery. It can be temporarily severe after parathyroidectomy in
patients with bone disease (postoperative bone hunger). The signs and
symptoms can include anxiety, depression, confusion, Chvostek's sign, and
Trousseau's sign, as well as circumoral or extremity tingling, tetany with
carpopedal spasms, or seizures. Treatment is as noted.

16. True statements about pituitary anatomy and physiology include:


A. The pituitary has dual embryonic origin: the anterior pituitary arises from
embryonic ectoderm; the posterior pituitary, from the diencephalon.
B. The hypophyseal portal system integrates function of the anterior and
posterior pituitary.
C. Adrenocorticotropin (ACTH), formed by posttranslational processing of
the precursor POMC, is normally controlled by hypothalamic CRF but may
be released by immune-related mechanisms.
D. Growth hormone (GH) directly stimulates longitudinal growth of the
skeleton and growth of muscles.
E. Cell types of the anterior pituitary are classified by their position in the
anterior pituitary and by their staining characteristics with histologic dyes.
Answer: AC
DISCUSSION: The anterior pituitary arises from Rathke's pouch (embryonic
ectoderm), and it includes the pars distalis, pars intermedia, and pars
tuberalis. The posterior pituitary arises from the diencephalon and includes
the neural stalk, infundibulum, and posterior lobe. The hypophyseal portal
system drains from the hypothalamus and integrates function of the
hypothalamus and anterior pituitary by carrying hypothalamus-derived
releasing factors to target cells in the anterior pituitary. ACTH may be
released during stress in response to interleukin-1, -2, and -6. Somatotrophic
actions of GH are indirect and are mediated by the insulinlike growth factors
formerly known as somatomedins. Current classification of anterior pituitary
cells is based on immunochemical identification of their secretory products
(i.e., corticotropes produce ACTH, lactotropes produce prolactin, and
thyrotropes produce thyroid-stimulating hormone [TSH]).

17. Antidiuretic hormone (ADH):


A. Is related to oxytocin, and both are released from the posterior pituitary in

conjunction with neurophysins.


B. Is released into the circulation by the posterior pituitary in response to a
rise in plasma osmolality above 285 mOsm. or a decrease in circulating blood
volume.
C. May be stimulated by catecholamines and inhibited by phenytoin, alcohol,
and lithium.
D. In excess, may produce a syndrome of euvolemic hyponatremia with
inappropriately concentrated urine that is responsive to free water restriction.
E. Deficiency causes prolonged polyuria and polydipsia and may be
diagnosed by a combination of high plasma osmolality and low urine
osmolality following water deprivation.
Answer: ABCDE
DISCUSSION: ADH and oxytocin are nineamino acid peptides derived
from a common ancestral peptide, vasotocin. ADH is released from the
posterior pituitary with neurophysin II in response to a rise in plasma
osmolality greater than 285 mOsm/kg. H 2O. by a 5% or greater decrease in
blood volume, and by catecholamines. The syndrome of inappropriate ADH
release (SIADH) produces euvolemic hyponatremia and is responsive to free
water restriction. Diabetes insipidus (DI) reflects a deficiency of ADH and
causes prolonged polyuria and polydipsia. Diabetes insipidus is diagnosed by
a combination of high plasma osmolality and low urine osmolality following
water deprivation. Exogenously administered ADH differentiates central
(ADH-responsive) DI from nephrogenic (ADH-unresponsive) DI.

18. Signs and symptoms of acute pituitary apoplexy include:


A. Severe headache.
B. Meningismus.
C. Vision loss.
D. Shock.
E. May be relieved by emergent transsphenoidal decompression of the sella

turcica.
Answer: ABCD
DISCUSSION: Acute pituitary apoplexy follows sudden hemorrhage into a
pre-existing pituitary tumor or following closed head trauma. Symptoms,
including headache, meningismus, and vision loss, are attributable to the
intracerebral blood. Pituitary insufficiency, as well as the accompanying
secondary adrenal insufficiency, may cause hypotension and shock. Other
manifestations may include DI and myxedema. Acute pituitary apoplexy is a
neurosurgical emergency that requires transsphenoidal decompression of the
sella turcica.

19. Prolactinomas of the pituitary:


A. Most often produce dysfunctional uterine bleeding in women.
B. Most commonly produce infertility in men.
C. When asymptomatic, are best treated surgically early in the microadenoma
stage.
D. May enlarge during pregnancy, requiring treatment with bromocriptine or
surgery.
E. Commonly occur in patients with MEN 2.
Answer: D
DISCUSSION: Pituitary prolactinomas are the most common pituitary tumor
and cause amenorrhea, galactorrhea, or both in women. Men usually have
macroprolactinomas and experience symptoms of a space-occupying lesion
of the sella (i.e., headache). A minority of men experience impotence and
infertility. Asymptomatic prolactinomas remain stable over time and require
observation only. Symptomatic prolactinomas require treatment, usually with
bromocriptine. Prolactinomas may enlarge during pregnancy and can produce
symptoms requiring treatment with either bromocriptine or surgery.
Prolactiomas are seen in as many as 40% of patients with MEN 1.

20. Hypercortisolism:
A. Is most often ACTH-dependent, owing to an ACTH-producing pituitary
adenoma.
B. Is best diagnosed by measurement of cortisol from a serum sample
collected at 8 A.M.
C. Is attributable to an adrenal source if the basal serum ACTH level is above
10 pg. per ml., if the hypercortisolism is suppressed by high-dose
dexamethasone, and if an adrenal tumor is visualized radiographically.
D. May be caused by small cell carcinoma of the lung, carcinoid tumors,
tumors of the endocrine pancreas, pheochromocytoma, or medullary thyroid
carcinoma (MTC).
E. In children is most often caused by adrenocortical neoplasia.
Answer: ADE
DISCUSSION: Cushing's syndrome is most often (80% to 90%) ACTHdependent and is most often due to an ACTH-secreting pituitary adenoma
(Cushing's disease). Some 10% to 20% of ACTH-dependent Cushing's
syndrome is caused by ectopic production of ACTH from small cell
carcinoma of the lung, carcinoid tumors, tumors of the endocrine pancreas,
pheochromocytoma, and MTC. ACTH-independent Cushing's syndrome
secondary to primary adrenal pathology occurs in 10% to 20% of cases in
adults and is the most common form in children. Measurement of cortisol in
two to three consecutive 24-hour collections of urine is the best screening test
for Cushing's syndrome: plasma levels of cortisol show marked variability,
and a single random level is not helpful in establishing the diagnosis. ACTHindependent Cushing's syndrome suppresses the pituitary and is diagnosed if
the basal serum ACTH level is suppressed below 5 pg. per ml., if the
hypercortisolism is not suppressed by high-dose dexamethasone, or if the
metyrapone stimulation test is negative. The dexamethasone suppression and
metyrapone tests do not, however, distinguish an adrenal from an ectopic

ACTH cause of Cushing's since both suppress the pituitary. Radiographic


evidence of an adrenal tumor must be supported by biochemical testing to
confirm that the adrenal is the primary cause of Cushing's syndrome.

21. Primary aldosteronism:


A. Produces a syndrome of diastolic hypertension, hypokalemia, and edema.
B. Is suggested by findings of serum potassium less than 3.5 mEq. per liter,
urinary potassium excretion greater than 30 mEq. per day, upright plasma
renin below 3 ng. per ml., and a plasma aldosterone concentrationplasma
renin activity ratio greater than 20:1.
C. Is most often due to an aldosterone-producing adrenal adenoma, which
may be distinguished from idiopathic adrenal hyperplasia by its sensitivity to
diurnal changes in ACTH and insensitivity to changes in posture.
D. May be diagnosed in hypertensive patients by demonstration of an adrenal
mass larger than 1 cm. on computed tomography alone.
E. Is best treated surgically if it is due to either aldosteronoma or idiopathic
adrenal hyperplasia.
Answer: BC
DISCUSSION: Primary aldosteronism is a syndrome of diastolic
hypertension and hypokalemia; edema typically is absent. The diagnosis
relies on demonstration of hypokalemia with inappropriate kaliuresis and
inappropriately elevated plasma aldosterone with suppression of plasma renin
following sodium loading. Primary aldosteronism most often is due to an
adrenal adenoma, which must be distinguished from idiopathic adrenal
hyperplasia since resection of the adenoma is nearly always curative, whereas
adrenal resection for idiopathic adrenal hyperplasia is curative less than 20%
of the time. CT evidence of adrenal tumor alone is inadequate to diagnose
aldosteronism, even in a hypertensive patient.

22. Adrenocortical carcinoma:


A. May be suspected in a patient with rapidly progressive Cushing's
syndrome and virilizing features or in asymptomatic patients with adrenal
tumors larger than 6 cm on CT.
B. Most often is diagnosed early in its course when disease is confined to the
adrenal gland.
C. Is differentiated from benign adrenocortical adenoma by tumor necrosis,
hemorrhage, and cellular features of large hyperchromatic nuclei and more
than 20 mitoses per high-power field.
D. Should be resected only if disease is localized to the adrenal gland;
otherwise treatment with mitotane is indicated.
E. Carries a poor prognosis: overall 5-year survival less than 25%.
Answer: AE
DISCUSSION: Patients with adrenocortical carcinoma often present with
rapidly progressive syndromes of combined adrenocortical hormone excess.
However, approximately half do not have syndromes of hormone excess but
present with abdominal pain, increased abdominal girth, weight loss, and
anorexia. Furthermore, incidentally discovered adrenal masses larger than 6
cm. harbor carcinoma in as many as 92% of cases. Adrenocortical carcinoma
is differentiated from adenoma by pathologic demonstration of either local
invasion or distant metastases. Most patients with adrenocortical carcinoma
present with locally advanced (stage III) or metastatic (stage IV) disease.
Prognosis for these patients is poor: 5-year survival is less than 25% in most
series. Patients with adrenocortical carcinoma should undergo primary
surgical resection or palliative surgical debulking of locally advanced,
metastatic, or even recurrent disease since response of these tumors to
medical therapy, including mitotane, is poor.

23. Addisonian crisis, or acute adrenocortical insufficiency:


A. Occurs only in patients with known adrenal insufficiency or in those

receiving long-term supraphysiologic doses of exogenous steroids.


B. Can mimic an acute abdomen with fever, nausea and vomiting, abdominal
pain, and hypotension.
C. May cause electrolyte abnormalities, including hypernatremia,
hypokalemia, hypoglycemia, and hypercalcemia, as well as eosinophilia on
peripheral blood smear.
D. Should be diagnosed with the rapid ACTH stimulation test before steroid
replacement is instituted.
E. May be effectively treated with intravenous stress-dose glucocorticoid
and mineralocorticoid replacement.
Answer: B
DISCUSSION: Addisonian crisis, or acute adrenal insufficiency, may be seen
following even mild illness in patients with suppression of the hypothalamicpituitary-adrenal axis. This suppression can be produced by as little as 1 week
of supraphysiologic stress dose steroids in the year before the stressful event.
Addisonian crisis is a medical emergency that requires prompt treatment
based on clinical suspicion. Clinical findings include fever, nausea and
vomiting, abdominal pain, and hypotension. Laboratory analysis may reveal
electrolyte abnormalities, including hyponatremia, hyperkalemia,
hypoglycemia, and hypercalcemia, as well as eosinophilia on peripheral
blood smear. The rapid ACTH test is diagnostic, but it should not delay
treatment with intravenous fluid resuscitation, glucose replacement, and highdose dexamethasone. Dexamethasone, not hydrocortisone, should be given
initially, since it does not interfere with subsequent determination of plasma
cortisol. Stress dose steroids are inadequate once adrenal crisis has occurred,
and exogenous mineralocorticoids are given when the patient resumes oral
intake.

24. Preparation for surgical removal of a pheochromocytoma includes:


A. Beta-adrenergic blockade followed by alpha-adrenergic blockade.

B. Hydration.
C. Alpha-adrenergic blockade, with or without beta-adrenergic blockade.
D. Preoperative Swan-Ganz monitoring in all patients.
E. Planning removal through an anterior, posterior, or laparoscopic approach
based upon tumor localization with CT, magnetic resonance imaging (MRI),
and/or 131I-MIBG.
Answer: BCE
DISCUSSION: Principles of preoperative management for
pheochromocytoma include preoperative alpha-adrenergic blockade using
phenoxybenzamine or phentolamine. Beta-adrenergic blockade with
propranolol is then used selectively in patients who develop tachycardia, have
a history of cardiac arrhythmia, or have primarily epinephrine-secreting
tumors. Beta-adrenergic blockade should be undertaken only after successful
alpha blockade is established. Patients with pheochromocytoma frequently
exhibit intravascular volume depletion, and careful hydration is mandatory.
Central venous pressure monitoring alone is helpful to guide hydration; more
intensive monitoring with a Swan-Ganz catheter is indicated for patients with
pre-existing heart disease. Formerly, the anterior approach was preferred for
adrenalectomy as it facilitated complete abdominal exploration and search for
extra-adrenal pheochromocytoma. Accurate preoperative localization with
CT, MRI, and 131I-MIBG has allowed selective use of the posterior, or even
the laparoscopic, approach for adrenalectomy.

25. Indications for surgical adrenalectomy include:


A. An adrenal mass larger than 6 cm.
B. Hypertensive patients with aldosteronism that is ACTH insensitive and
posture sensitive and who also have multiple adrenal nodules on CT.
C. Cushing's syndrome secondary to adrenal neoplasms or to persistent
ectopic ACTH syndrome when the primary tumor is inoperable.
D. Pheochromocytoma in adults and children.

E. Congenital adrenal hyperplasia secondary to 21-hydroxylase deficiency.


Answer: ACD
DISCUSSION: Adrenalectomy is indicated for the management of adrenal
masses larger than 6 cm. on CT since tumors of this size harbor carcinoma in
more than 90% of cases. Surgical adrenalectomy is also the treatment of
choice for primary adrenal causes of Cushing's syndrome. Surgical
adrenalectomy may be considered as an alternative to medical adrenalectomy
with metyrapone, aminoglutethimide, or mitotane in patients with ectopic
ACTH syndrome when treatment of the primary tumor is unsuccessful.
Patients with aldosteronism that is ACTH sensitive and posture insensitive
likely have an adrenal adenoma that is curable by adrenalectomy.
Aldosteronism that is ACTH insensitive and posture sensitive is likely caused
by idiopathic adrenal hyperplasia, which is best managed medically with
spironolactone, triamterene, amiloride, or nifedipine. Adrenalectomy
following preoperative alpha-adrenergic blockade, with or without betaadrenergic blockade, and hydration is the treatment of choice of all
pheochromocytomas. Adrenalectomy is not indicated in the management of
any of the congenital adrenal hyperplasias.

26. Incidental adrenal masses:


A. May be seen in as many as 10% of abdominal CT studies.
B. Most commonly represent pheochromocytoma; adrenocortical adenoma,
adrenocortical carcinoma, and metastases from other primary cancers occur
less frequently.
C. May represent adrenocortical carcinoma if greater than 6 cm. in diameter.
D. Should be routinely evaluated by measurement of 24-hour urine levels of
catecholamines and their metabolites, cortisol, and aldosterone plus fineneedle aspiration.
E. Should be resected if biochemically active, if greater than 6 cm., or if they
grow over six months' follow-up.

Answer: CE
DISCUSSION: The incidental adrenal mass is seen in as many as 1.3% of
abdominal CT scans performed for other reasons. Adrenocortical adenomas
are most common, followed by adrenocortical carcinoma, metastases from
other primary cancers, and pheochromocytoma. Biochemical evaluation must
weigh the prevalence of adrenal neoplasms against the consequences of a
missed life-threatening diagnosis, as in pheochromocytoma. All adrenal
masses should be evaluated for pheochromocytoma with measurement of 24hour urine catecholamines and their metabolites. Aldosterone and cortisol
measurement are indicated if clinical features suggest aldosteronism or
Cushing's syndrome. Fine-needle aspiration of adrenal masses is indicated for
clearly cystic lesions or if metastasis is suspected based on the presence of
another known primary. Fine-needle aspiration is not routinely indicated in
the evaluation of adrenal lesions and is contraindicated until
pheochromocytoma is definitively excluded. Adrenal lesions should be
resected if they are functional, are larger than 6 cm., or have enlarged during
follow-up.

27. Which of the following statements is true about the synthesis of thyroid
hormone and its physiology?
A. The iodine utilized in hormone synthesis is derived principally from
dietary sources.
B. The role of thyroid-stimulating hormone (TSH) in thyroid physiology is
limited to regulation of the release of thyroid hormone in plasma.
C. Enough thyroxine (T 4) is stored in the normal thyroid to provide a
euthyroid state for 3 weeks despite absence of iodine intake.
D. The regulation of thyroid function involves pituitary, but not
hypothalamic, input.
Answer: AC

DISCUSSION: Iodine is necessary for the synthesis of thyroid hormone, and


approximately 200 to 500 mg is ingested daily. Most of it is absorbed from
the small intestine and is cleared from the plasma by secretion of thyroid
hormone. TSH is required for the normal production and secretion of thyroid
hormone. It is generally accepted that TSH also has a major role in thyroid
growth. The thyroid gland has a storage reserve of approximately 3 weeks.
The principal regulatory mechanisms of the thyroid gland are the
hypothalamic-pituitary-thyroid control system and the intrathyroidal
regulatory systems. TRH is produced by the superoptic and paraventricular
nuclei within the hypothalamus and passes down their axones. Following
secretion into the hypophyseal portal blood systems TRH passes to the
pituitary and induces stimulation of TSH secretion.

28. Correct statements about thyroid function tests include which of the
following?
A. Contraceptive pills and pregnancy increase the amount of thyroxinbinding globulin (TBG), and, consequently, the total T 4 level.
B. Anticonvulsive medications and chronic debilitating illnesses decrease the
amount of TBG and, consequently, the total T 4 level.
C. Intravenous pyleography can lower the rate of active iodine uptake by the
thyroid.
D. A triiodothyronine (T 3) suppression test that demonstrates
nonsuppressibility of thyroid function is compatible with the diagnosis of
Graves' disease, toxic adenoma, or functioning carcinoma.
E. An increased serum cholesterol level in a hypothyroid patient indicates a
thyroid cause.
Answer: ABCD
DISCUSSION: Hormone binding proteins are the principal intravascular
factors influencing total hormone concentration. Various factors may cause
changes in the concentration of TBG. Contraceptive pills and pregnancy

increase the amount of TBG, while anticonvulsive medications and chronic


debilitating disease may decrease the amount of TBG. Administration of
excess amounts of iodine may lower the rate of active iodine uptake in the
thyroid gland. The thyroid suppression test is based on the principle that
administration of thyroid hormone does not suppress the patient's thyroid
function when normal homeostatic mechanisms are disrupted. Thyroid
function is nonsuppressible in hyperthyroidism or in the presence of thyroid
hormonesecreting tumors. Serum cholesterol, deep tendon reflex time, and
cardiovascular functions can be measured to reflect hypothyroidism and
hyperthyroidism but do not give an indication of the etiology of
hypothyroidism.

29. Hyperthyroidism can be caused by all of the following except:


A. Graves' disease.
B. Plummer's disease.
C. Struma ovarii.
D. Hashimoto's disease.
E. Medullary carcinoma of the thyroid.
Answer: E
DISCUSSION: Common types of hyperthyroidism include diffuse toxic
goiter (Graves' disease, named after the Dublin physician Robert Graves who
described it in 1835 but known since its original description by Parry in 1786
and described by von Basedow in 1840) and toxic adenoma or toxic
multinodular goiter (Plummer's disease). Uncommon causes include
thyrotoxicosis factitia, functioning metastatic thyroid carcinoma,
trophoblastic tumors that secrete human chorionic gonadotropin (having
thyroid-stimulating properties), inappropriate secretion of thyrotropin by
pituitary tumors, struma ovarii, iodide-induced hyperfunction, and thyroiditis.

30. Which of the following is true about the use of radioiodine to treat
hyperthyroidism?
A. If hyperthyroidism is secondary to radioiodine use, it will occur within 2
years of treatment.
B. There is a markedly increased risk of future thyroid cancer following
radioiodine therapy.
C. The risk of leukemia following radioiodine therapy is approximately 10%.
D. Mutation abnormalities occur in 15% of fetuses in utero following internal
treatment of the mother with radioiodine during pregnancy.
E. Radioiodine may pass through the placenta and lactating breast to produce
hypothyroidism in a fetus or infant.
Answer: E
DISCUSSION: Hypothyroidism secondary to radioiodine therapy increases
in frequency with time to 85% after 5 years. There has been no demonstration
of an increase in malignancies following radioiodine therapy. Radioactive
iodine can cross the placenta and lactating breast to produce hypothyroidism
in a fetus or a nursing infant and is, therefore, contraindicated.

31. Arrange the following complications of thyroid surgery (bilateral subtotal


thyroidectomy) in decreasing order of incidence in patients with Graves'
disease.
A. Laryngeal nerve paralysis.
B. Hypoparathyroidism.
C. Hypothyroidism.
D. Recurrent hyperthyroidism.
Answer: CDAD
DISCUSSION: The incidence of recurrent disease is inversely related to the
incidence of hypothyroidism and is 1% to 5%. Within 1 to 2 years,
hypothyroidism may develop in 5% to 50% of patients (with a slight

additional increase in subsequent years). The associated morbidityrelated


primarily to damage to the recurrent laryngeal nerves and parathyroid glands
is estimated to be 0.5% to 3.0%.

32. The most common cause of goitrous hypothyroidism in adults is:


A. Graves' disease.
B. Riedel's thyroiditis.
C. Hashimoto's disease.
D. de Quervain's thyroiditis.
Answer: C
DISCUSSION: Hashimoto's disease was first described in Japan by Hakaru
Hashimoto in 1912 and is the best-known of the immunologic thyroid
diseases. It is the most common cause of goitrous hypothyroidism in adults
and of sporadic goiter in children. The incidence is 0.3 to 1.5 cases per 1000
population per year and it is 10 to 15 times more common in women than in
men, with the highest incidence in the group aged 30 to 50 years.

33. Therapy for Hashimoto's disease includes:


A. Radioiodine.
B. Antithyroid medications.
C. Subtotal thyroidectomy.
D. None of the above.
Answer: D
DISCUSSION: There is no specific treatment for Hashimoto's disease.
Patients are usually followed medically, and replacement therapy with T 4 is
begun in patients with hypothyroidism that is symptomatic or associated with
a goiter that is causing pressure symptoms. Early initiation of thyroid
hormone therapy has been recommended by many to prevent further thyroid

enlargement and reduce the risk of myxedema, especially in postpartum


patients. Surgical reduction of goiter should be performed if severe pressure
symptoms that have not responded to corticosteroid therapy are present. This
usually consists of subtotal thyroidectomy. Biopsy to rule out malignancy in
nodules suspicious for thyroid carcinoma (usually papillary) or lymphoma is
indicated. If carcinoma is suspected, lobectomy should be performed, and if
frozen section demonstrates carcinoma, subtotal or total thyroidectomy
should be performed.

34. Indications for surgical thyroidectomy for Graves disease include which
of the following?
a. Ocular involvement
b. Symptomatic large goiter
c. Women of childbearing age
d. Concomitant thyroid nodule
e. All of the above
Answer: b, c, d
Antithyroid drugs are the initial therapy in most patients with Graves
disease, either as a definitive therapy or in preparation for 131I therapy or
surgical ablation. Because of the high failure rate of long-term treatment with
thionamides, the use of these drugs as definitive treatment has decreased.
Drawbacks include the important 0.5% incidence of agranulocytosis and a
recurrence rate as high as 43% during the first year when the drug is stopped.
Five years after treatment only 25% of patients remain in remission. Unlike
other definitive treatments, hypothyroidism does not occur as a result of
thionamide treatment if an appropriate dosage is used. 131I has been used as
definitive treatment for patients with Graves disease for many years with
predictable and long-lasting good results in most patients. It has few, if any,
serious side effects. It is ablative to the thyroid gland and hypothyroidism is a

nearly inevitable result of effective therapy, although it may take years to


become clinically apparent. About 70% of patients treated with 131I are
hypothyroid within 10 years of treatment. The risk of recurrence of
hyperthyroidism after an initial response is less than 5%. Most adult patients
in the United States are treated with 131I as definitive treatment for Graves
disease. Exceptions are women in the childbearing years where a subsequent
fetus would be affected, patients with concomitant thyroid nodules where
carcinoma is a concern, those with extremely large glands and, increasingly,
those who are opposed to 131I therapy. Thyroidectomy is an important
alternative in selected patients with Graves disease. Although controversial
in the past, it now appears that ocular involvement does not respond more
favorably to thyroidectomy than to 131I ablation.

35. Which of the following statements regarding anatomic relationships of


the thyroid gland are true?
a. The middle thyroid artery is intimately related to the superior laryngeal
nerve
b. The superior thyroid artery is usually the first branch of the external carotid
artery
c. Thyroidea ima arteries are found in approximately 20% of individuals
d. The parathyroid glands may lie within the pretracheal fascia
Answer: b, d
The thyroid gland is a vascular organ supplied by four main arteries: two
superior and two inferior. The superior thyroid artery usually arises as the
first branch of the external carotid artery just above the bifurcation of the
common carotid artery. The superior thyroid artery descends medially on the
surface of the inferior pharyngeal constrictor muscle to divide into an anterior
and posterior branch at the apex of the thyroid lobe on its anteromedial
surface. Its relation to the external branch of the superior laryngeal nerve is

important during thyroid lobectomy. The inferior thyroid arteries usually arise
from the thyrocervical trunks and ascend behind the carotid sheath before
passing downward and medial to enter the thyroid gland at its middle portion.
There are no arteries directly entering the lower poles from below with the
exception of a thryoidea ima artery that may replace an absent inferior artery.
Thyroidea ima arteries arise from either the innominate artery or aorta in 1%
to 4% of individuals, entering the lower surface of the isthmus after coursing
on the trachea.
The pretracheal fascia is referred to as the thyroid sheath and it varies in
consistency and completeness among individuals. Posteromedially, it is
usually condensed and firmly attaches the thyroid gland to the upper two or
three tracheal rings and the cricoid cartilage. The superior parathyroid gland
may lie between the sheath and the thyroid capsule, within the sheath,
posterior to it in a potentially open plane, or occasionally anterior to the
sheath. The inferior parathyroid gland likewise may be within the sheath,
particularly when the gland is adherent to the lower pole of the thyroid.

36. Radioactive iodine is effective treatment for metastatic lung disease for
which of the following thyroid neoplasms?
a. Hrthle cell carcinoma
b. Papillary carcinoma
c. Follicular carcinoma
d. Medullary carcinoma
e. Anaplastic carcinoma
Answer: b, c
Radioactive iodine is used only in patients who have differentiated thyroid
carcinomas. It is of no value in the treatment and follow-up of patients with
Hrthle cell, medullary or anaplastic carcinomas. Most papillary carcinomas
are capable of taking up radioactive iodine. Most papillary carcinomas in

patients under 50-years of age do so, providing that the patient has had a total
thyroidectomy and there is no normal thyroid tissue to compete for the 131I.
About 20% of all papillary carcinomas do not trap sufficient iodine for
imaging or therapy. These are usually patients with papillary carcinoma
variants: a tall cell variant of papillary carcinoma, insular carcinoma, or clear
cell carcinoma.
Nearly all metastatic follicular carcinomas retain the ability to trap 131I
sufficiently for imaging and for therapy. Even well-differentiated papillary
and follicular carcinoma cannot compete successfully for 131I with normal
thyroid tissue and unless this has been removed or subsequently ablated with
an initial dose of 131I, many metastases cannot be detected or treated.

37. The types of thyroiditis that can cause abnormalities of surgical


significance are which of the following?
a. Chronic lymphocytic thyroiditis (Hashimoto disease)
b. Riedel struma
c. Acute (viral) thyroiditis
d. Granulomatous (subacute) thyroiditis
Answer: a, b, d
There are three types of thyroiditis that can cause thyroid abnormalities of
surgical significance. The most common is chronic lymphocytic (Hashimoto)
thyroiditis, an autoimmune disease that can occur in any age group.
Occasionally, Hashimoto disease causes unilateral thyroid enlargement that
simulates malignancy. The rarest form of thyroiditis is Riedel struma which
can mimic a diffuse thyroid carcinoma because of the fibrotic infiltrative
process that results. Hashimoto thyroiditis is associated with reduced
functional capacity of the thyroid which increases TSH secretion, and a goiter
develops. Because of the associated fibrosis, a nodular goiter or neoplasm is
suggested. Thyroidectomy may be indicated for treatment of a solitary

nodule, particularly if it is cold, suspicious, definitely malignant or solid, and


fine needle aspirate is indeterminate.
Granulomatous, DeQuervain, and subacute thyroiditis are terms that refer to a
disease that usually occurs in young women within weeks of an upper
respiratory or other viral infection. The disease is usually self-limited, but
may persist for several months longer. In unusual patients, the disease may be
confined to one lobe and result in a firm, slightly tender mass suggesting
carcinoma. Lobectomy may be indicated to rule out the presence of
malignancy. Total thyroidectomy may be considered for persistent, painful
thyroiditis after months of steroid therapy have failed.
Goiter with a woody or fibrous component involving the adjacent strap
muscles and carotid sheaths is referred to as Riedel struma. It is rare and the
cause is not known. It is associated with other types of fibrotic processes
including retroperitoneal fibrosis, sclerosing cholangitis, and fibrosing
mediastinitis. Although considered self-limited, the process may be
associated with considerable morbidity as a result of localized pain and
compression of adjacent structures such as the airway. Occasionally,
tracheostomy is required. Airway compression may also require open biopsy
resection of the isthmus with as much as the fibrosis as possible without
endangering the recurrent laryngeal nerves.

38. The principal blood supply to the parathyroid glands is which of the
following?
a. Superior thyroid arteries
b. Inferior thyroid arteries
c. Thyroidea ima arteries
d. Parathyroid arterial branches directly from the external carotid artery
e. Highly variable
Answer: b

The principal blood supply to both parathyroid glands is the inferior thyroid
artery. Parathyroid glands invariably have a single end artery supplying them,
and if the main trunk of the inferior thyroid artery is ligated during
thyroidectomy, there is no collateral blood supply to maintain their viability.
It is preferable to divide only the branch of the inferior thyroid artery medial
to those that supply either of the parathyroid glands. This requires individual
clamping of smaller vessels under the thyroid sheath as these vessels
penetrate into the thyroid capsule. Ligation of the main trunk of the inferior
thyroid artery was commonly used for bilateral subtotal thyroidectomy in the
past. It did not routinely cause hypoparathyroidism only because enough
collateral blood supply was maintained to each end artery to one or more
parathyroid glands. This is to be avoided.

39. Which of the following statements regarding papillary thyroid carcinoma


are true?
a. Seventy to 80% of new cases of thyroid carcinoma in the United States are
of the papillary type
b. Total ipsilateral lobectomy and isthmus resection are adequate therapy for
minimal thyroid carcinoma
c. Microscopic evidence of multicentric disease is present in 70% to 80% of
cases
d. Nearly all patients less than 15 years of age have metastatic disease in local
lymph nodes
Answer: a, b, c, d
Seventy to 80% of the 11,000 new patients with thyroid carcinoma diagnosed
annually in the United States have papillary carcinoma. Papillary carcinomas
of the thyroid include minimal thyroid carcinoma, intrathyroidal, and
extrathyroidal (invasion through the true thyroid capsule) disease. Minimal
thyroid carcinoma refers to those papillary carcinomas that are less than 1 cm

in diameter and not associated with any clinically apparent lymph node
metastases. In contrast to clinically significant papillary carcinomas, these are
common and are found in 2% to 13% of adult thyroid glands serially
sectioned after autopsy studies of individuals who have died from other
causes. For tumors between 0.5 and 1 cm, a total lobectomy and isthmus
resection are satisfactory treatment.
Most clinically significant papillary carcinomas are 1 to 4 cm in diameter and
are contained within the thyroid capsule. Multicentricity is relatively
common and can be found on gross sectioning of the thyroid gland in 20% to
30% of cases. Furthermore, after serial sectioning of the entire thyroid gland
in patients with papillary carcinoma, microscopic foci are found in 70% to
80%. Local cervical lymph node metastases are found in about 30% of all
patients with papillary carcinoma. The presence of lymph node metastases
does not correlate as closely to the size of the tumor as it does to the age of
the patient. The younger the patient, the greater the likelihood of metastatic
lymph node involvement. Nearly all patients under 15 years of age have
involved metastatic lymph nodes. The presence or absence of lymph node
metastases in patients with intrathyroidal primary papillary carcinomas does
not appear to have an appreciable effect on long-term survival if distant
metastases are not present at the time of initial treatment.

40. A 30-year-old female presents for evaluation of a palpable thyroid nodule.


Technetium-99m (99mTc) scan demonstrates a single cold nodule. The
differential diagnosis includes which of the following?
a. Carcinoma
b. A nonfunctioning adenoma
c. A thyroid cyst
d. A colloid nodule
e. An autonomous nodule
Answer: a, b, c, d

Radioisotope scanning measures the functional activity of the thyroid gland


and maps its correlation with physical findings. For routine scanning, 131I
scanning has been replaced by either 123I or 99mTc. Because of the lower
radiation exposure, cost and easy use, 99mTc is preferentially used. The
differential diagnosis of a solitary nonfunctioning nodule includes carcinoma,
colloid nodule, nonfunctioning adenoma, and cyst. The prevalence of
carcinoma ranges from 5% to 20% in cold nodules and such lesions require
further evaluation. If the scan detects other nonfunctioning areas in addition
to the palpable nodule, the gland is at low risk for carcinoma because most
patients with this finding have a multinodular goiter. A solitary, discrete area
of increased activity is more often found in a young patient with an otherwise
normal thyroid gland. Multiple hot spots are typically found in an older
patient with a multinodular goiter.
A functioning solitary nodule that is independent of TSH is considered an
autonomous nodule and can be the cause of hyperthyroidism. It is hot on
scintigraphic scan. The presence of a cold nodule is insufficient information
to determine a treatment plan. Although nearly all carcinomas are cold, most
cold nodules are benign. Fine needle aspiration cytology is considered the
most reliable means of evaluation for the diagnosis of thyroid nodules that
are nonfunctional or hypofunctional by nuclide scan.

41. Which of the following pharmacologic agents can be used in the


treatment of thyrotoxicosis to block the production of thyroid hormone?
a. Propylthiouracil
b. Propranolol
c. Methimazole
d. Carbimazole
e. Iodine
Answer: a, c, d, e

A number of substances interfere with normal production of thyroid hormone


by blocking one of the relevant enzymatic steps. Iodine was the first effective
drug to be used in the treatment of thyrotoxicosis. It can block the
organification and coupling steps in thyroid hormone synthesis as well as
prevent the release of thyroid hormone. Furthermore, iodine in large doses
probably inhibits the ability of TSH to stimulate cyclic AMP release at the
follicular cell membrane.
Commonly used antithyroid drugs are propylthiouracil (PTU), methimazole
(Tapazole) and carbimazole. Carbimazole is widely used in Great Britain,
only PTU and methimazole are commonly used in the United States. PTU
interferes with the incorporation of iodine into the tyrosine residues of
thyroglobulin, thus preventing oxidation of iodide to iodine. It also inhibits
the peripheral conversion of T4 to T3. Although both PTU and methimazole
are thionamides, methimazole does not have this peripheral effect, making
PTU the preferred drug for patients with thyroid storm. About 3% of patients
taking PTU demonstrate at least one side effect during the first 3 months of
therapy whereas the prevalence with methimazole is about 7%. These range
from minor skin rashes to agranulocytosis that can be irreversible.
b-adrenergic antagonists have been used in the treatment of hyperthyrodism.
Propranolol is the most widely used of these drugs. However, b-blockade
does not alter thyroid function per se. Rather, its effect is to provide
symptomatic relief of hyperthyroidism because of interference with the action
of thyroid hormones at the cellular level.

42. A 45-year-old woman has a solitary, nonfunctioning thyroid nodule and


fine needle cytology is nondiagnostic. Which of the following is the initial
surgical procedure of choice?
a. Total extracapsular thyroidectomy
b. Subtotal thyroid lobectomy and resection of the isthmus

c. Total extracapsular thyroid lobectomy, resection of the isthmus, and


modified unilateral neck dissection
d. Total extracapsular thyroid lobectomy and resection of the isthmus
Answer: d
Total extracapsular thyroid lobectomy and isthmus resection is the procedure
of choice when a decision has been made to surgically remove a thyroid
nodule. The entire lobe with the isthmus is submitted for frozen-section
pathologic examination if fine needle aspirate has not already resulted in a
definitive diagnosis of carcinoma. In performing total lobectomy, both
parathyroid glands are carefully preserved with their blood supply. This is
done in the event that total thyroidectomy is necessary if either the frozen or
permanent histologic sections confirm the presence of thyroid carcinoma.
Total lobectomy offers the best opportunity for accurate histologic diagnosis
and is associated with the lowest incidence of complications when the need
for reoperation is considered. In one experience, 800 consecutive cases of
total unilateral lobectomy were performed for benign or malignant nodules
suspected of cancer, and no permanent recurrent laryngeal nerve palsies
occurred. Primary total lobectomy is safer than a partial lobectomy followed
by resection of the residual lobe after a delayed diagnosis of malignancy.
Reoperation to complete a lobectomy is associated with a greater risk to both
recurrent laryngeal nerve and the parathyroids on the ipsilateral side.
Although there is controversy as to whether a total lobectomy and isthmus
resection or a total thyroidectomy is the best definitive operation for
unilateral papillary carcinoma, a subtotal lobectomy is universally considered
an inadequate operation. A definitive cancer operation can be accomplished
with one procedure in 80% of the cases when a skilled thyroid pathologist is
available for frozen-section interpretation.

43. Which of the following statements regarding fine needle aspiration


cytology of a thyroid nodule are true?

a. It differentiates neoplastic and nonneoplastic nodules in most cases


b. It does not allow differentiation of papillary, medullary and anaplastic
carcinoma
c. It cannot differentiate malignant and benign follicular or Hrthle cell
neoplasms
d. It is not recommended when a patient has a history of head and neck
radiation
Answer: a, c, d
When interpreted by a skilled cytologist, fine needle aspiration is highly
accurate and is considered the preferred method of selecting patients with
thyroid nodules for surgery. Nearly 80% of patients with thyroid nodules
were spared surgical exploration as a result of such studies in one report.
Because of the risk of false-negative diagnoses (10%), advocates of this
technique emphasize the importance of clinical judgment in addition to the
cytologic study in selecting operative candidates.
In most cases, fine needle aspiration cytology enables the pathologist to
distinguish nonneoplastic from neoplastic nodules and to identify the type of
malignant tumor. Papillary, medullary, and anaplastic carcinoma all have a
typical cytologic appearance. Cytologic studies cannot differentiate malignant
from benign follicular or Hrthle cell neoplasms. In this case, a definitive
diagnosis depends on histologic examination of the entire excised tumor.
Fine needle aspiration has dramatically reduced the number of diagnostic
surgical operations for benign lesions in centers where it is used extensively.
In patients with thyroid nodules and a history of previous head and neck
radiation, operation is generally recommended regardless of cytologic
findings. In these patients, both benign and malignant lesions may develop
and the chances of sampling error are considerable.

44. Hyperthyroidism results from all of the conditions noted below. Of the

following which commonly require surgical management?


a. Graves disease
b. Struma ovarii
c. Functioning metastatic thyroid carcinoma
d. Toxic diffuse goiter
e. Single toxic thyroid nodule
Answer: a, d, e
Hyperthyroidism is associated with clinical manifestations related to an
excess of thyroid hormone. There are three causes of primary concern to the
surgeon. Graves disease, or toxic diffuse goiter, is most common, accounting
for more than 80% of all patients with hyperthyroidism. The other two
relevant causes of hyperthyroidism are toxic nodular goiter and a single toxic
nodule. Common causes of hyperthyroidism that rarely require surgery are
postpartum thyroiditis, iodine-induced hyperthyroidism, self-administered or
iatrogenic hyperthyroidism, struma ovarii, functioning metastatic carcinoma
and several rare forms of thyroiditis.

45. Which of the following statements regarding medullary carcinoma of the


thyroid are true?
a. Approximately 75% of all cases are hereditary
b. The overall 10-year survival rate is less than 10%
c. Medullary carcinoma of the thyroid is associated with both multiple
endocrine neoplasia IIa (MEN IIa) and multiple endocrine neoplasia IIb
(MEN IIb) syndromes
d. Prophylactic total thyroidectomy is recommended for MEN IIa and MEN
IIb patients after the age of 10 years
Answer: c

Medullary carcinoma of the thyroid (MCT) accounts for about 7% of all


malignant tumors of the thyroid. It is familial in 20% to 30% of all cases and
its secretion of a biologic marker (calcitonin) allows detection of its presence
with the tumor is too small to palpate. MCT appears in three clinical settings.
The first is a sporadic tumor, usually in patients 30 years or older. Second,
MCT occurs as a component of the MEN IIa syndrome with or without
adrenal medullary disease (pheochromocytoma) or hyperparathyroidism. A
family history of thyroid carcinoma with or without pheochromocytoma is
invariably present. MCT in the MEN IIa syndrome is always bilateral and
multicentric and arises from C-cell hyperplasia. MCT usually does not
develop before age 12 and is almost always clinically apparent before age 30.
Lastly, MCT is found as a component of the MEN IIb syndrome without or
without bilateral adrenal medullary disease and always with the facies and
autonomic nervous system dysplasia expressed as ganglioneuromatosis from
the lips to the anus. These patients often have a marfanoid habitus and
skeletal deformities as well. The MEN IIb syndrome occurs as a sporadic
mutation, but its familial occurrence is becoming more common because
patients are surviving now long enough to reproduce. MCT or its precursor,
C-cell hyperplasia, develops by age 2 in MEN IIb and is always bilateral and
multicentric. As a result of its early appearance and late detection, the disease
has usually been more advanced when treatment has been instituted. Minimal
treatment of MCT is total thyroidectomy. The overall prognosis for 10-year
survival is approximately 50%, although the tumor growth rate in individual
patients has shown great variability. Patients with the MEN IIb disease
require total thyroidectomy as soon as the syndrome is recognized, preferably
by the age of 2 years. In familial cases, the characteristic findings are
sufficient even without calcitonin testing to justify operation. If the diagnosis
is not made until adolescence or later, both central compartment and lateral
node involvement require neck dissection for definitive treatment.

46. Which of the following statements regarding thyroid physiology are true?

a. Normally about 20% of T3 is secreted directly from the thyroid gland


b. The thyroid gland is the only endogenous source of T4
c. Excess thyroid hormone results in an increase in the number of ATPdependent sodium pumps on the cell membrane
d. The majority of thyroid hormone in circulating plasma is bound to
albumin, thus limiting the availability of the metabolically active form
Answer: a, b, c
The thyroid is the only endogenous source of T4, whereas most T3 is
produced by the peripheral conversion of T4. This takes place in the liver,
muscle, kidney and anterior pituitary. Under normal circumstances, only
about 20% of T3 is secreted directly from the thyroid gland. In some thyroid
diseases (e.g., Graves disease and toxic nodular goiter), the proportion of T3
secreted directly by the thyroid may be markedly increased.
Once thyroid hormones are released into the circulation, they are bound to
thyroid-binding globulin (85%), albumin (10%) and transthyretin
(prealbumin). These binding proteins allow the thyroid hormones to remain
soluble in plasma, contributing to systemic distribution to various target-cell
populations. A limited amount of thyroid hormones circulates freely in the
plasma in metabolically active form (free T4, free T3).
At the cell membrane, an excess of thyroid hormone results in an increased
number of ATP-dependent sodium pumps, thus increasing resting energy
expenditure and oxygen consumption. Thyroid hormone also facilitates the
transport of glucose and amino acids across the cell membrane. In addition,
T3-induced mitochondrial oxidation of substrate results in increased ATP
production.

47. The definitive treatment of choice for toxic multinodular goiter is?
a. Total thyroidectomy

b. Bilateral subtotal thyroidectomy


c. Unilateral total lobectomy on the side of dominant disease
d. 131I treatment
Answer: b
Hyperthyroidism in the patient with a multinodular goiter usually develops in
women after age 50 but is seen occasionally in younger patients. Most
patients have had a nontoxic nodular goiter for many years. Preferred
treatment for most patients with toxic multinodular goiters is thyroidectomy
after adequate preparation renders the patient euthyroid. 131I may be an
alternative in selected poor-risk patients with goiters that are not causing
airway compression. Although 131I can be used to treat the hyperthyroidism,
larger and often repeated doses of 131I may be required. 131I does not
significantly reduce the goiter size and may, because of radiation-induced
thyroiditis, cause acute enlargement. This may be hazardous in the patient
with some degree of preexisting airway compression. Any airway symptoms,
particularly in patients with substernal goiters, should be considered strong
contraindications to the use of 131I.
Standard surgical treatment of toxic nodular goiter has consisted of bilateral
subtotal thyroidectomy. Remnant size is not as important as the excision of
all autonomous nodules. Because thyroid replacement or suppression is used
routinely to prevent recurrence of goiter when a subtotal resection is done,
the risk of hypothyroidism is not a consideration in determining remnant size.
Alternative procedures are total lobectomy with isthmus resection and
contralateral subtotal lobectomy, or total thyroidectomy. The latter is not
demonstrably superior and may have more technical complications.

48. A 50-year-old male has undergone an ipsilateral thyroid lobectomy and


isthmus resection for what appeared on frozen section to be a benign nodular
lesion 2.0 cm in diameter. Seventy-two hours later the final pathology returns
and the diagnosis is a high-grade angioinvasive follicular carcinoma. What do

you recommend?
a. 99mTc bone scan to rule out occult bone metastases
b. Ipsilateral radical neck dissection
c. Total thyroidectomy
d. Observation with sequential 131I scans every 3 months
Answer: c
Follicular carcinoma accounts for about 10% of all new carcinomas of the
thyroid. Most patients are those with minimal invasion of the capsule or
vessels within the neoplasms. Such tumors are seldom diagnosed definitively
by either needle aspiration cytology or by frozen-section diagnosis at the time
of lobectomy. Most frequently, the diagnosis is made after the study of
permanent sections. Microinvasive encapsulated follicular carcinomas are
rarely associated with metastatic lymph nodes and distant metastases
involving bone are also rare at the time of diagnosis.
Angioinvasive follicular carcinomas are usually large and frequently show
venous invasion of perithyroidal and lateral neck veins at the time of
diagnosis. They may have already metastasized to different sites, most
frequently the bone. These tumors are often diagnosed at the time of
operation because of these characteristics. Most patients younger than 40 do
well, but patients older than 50 years have a guarded prognosis. Follicular
carcinomas of the thyroid are treated by total thyroidectomy. Lymphatic
dissections are not usually required because only about 5% of all patients
have lymphatic involvement. The most effective therapy for bone or
pulmonary metastases is radioactive iodine. The controversy in the surgical
management of follicular carcinoma is whether patients with low-grade
encapsulated neoplasms diagnosed after total lobectomy by permanent
section evaluation should be treated with total thyroidectomy. In patients with
smaller follicular neoplasms found malignant as determined by
microinvasion of the capsule, completion thyroidectomy is usually not done;
instead patients are observed closely after performing a 99mTc bone scan to

rule out occult bone metastases. Patients with large or high-grade


angioinvasive lesions require total thyroidectomy.
49. In 1990 a National Institutes of Health Consensus Dev
pment Conference reviewed the available evidence regarding the management of
asymptomatic primary hyperparathyroidism. The panel concluded that surgical
intervention was indicated for which of the following indications?

a. Age less than 50 years of age


b. Reduced creatinine clearance

c. Presence of kidney stone(s) (as detected by abdominal radiograph)


d. Substantial reduced bone mass as determined by direct measurement
e. Markedly elevated 24-hour urinary calcium excretion
Answer: a, b, c, d, e
All of the indications listed above are considered indications for operative
treatment of asymptomatic patients with primary hyperparathyroidism. Their
conclusions regarding operative indications are summarized in Table 57-8.
The NIH Consensus Development Conference mandated close (every 6
months) follow-up for patients with known primary hyperparathyroidism not
treated by operation. In addition, surgery was agreed upon for those patients
in whom medical surveillance was neither desirable nor suitable, such as
when the patient requests surgery, consistent follow-up is unlikely, coexistent illness complicates management, or if the patient is young (< 50 year
of age). In one recent study of a group of 142 asymptomatic patients followed
without operation, after 10 years, more than 20% of the patients had required
surgery for an increase in serum calcium to greater than 11 mg/dL or for
specific complications attributable to the disease. Another 20% were lost to
or declined follow-up. The remainder either died of unrelated causes or had
persistent asymptomatic disease. This remains an area of considerable

controversy. The best available recommendations are derived from this


October 1990 NIH Consensus Development Conference. 50. Which of the
following statements regarding medullary carcinoma of the thyroid are true?
a. Bilateral medullary carcinoma of the thyroid occurs in every patient with
multiple endocrine neoplasia 1, 2a or 2b b. Medullary carcinoma of the
thyroid accounts for approximately one-third of all thyroid malignancies c.
20% of medullary carcinoma of the thyroid cases are familial d. Provocative
testing with pentagastrin and calcium infusion for family members at risk for
medullary carcinoma of the thyroid is diminished in importance by the recent
development of specific genetic testing Answer: a, c, d Medullary thyroid
carcinoma accounts for about 10% of all thyroid malignancies, and 20% of
cases occur in the familial setting of MEN 2a, MEN 2B, or familial nonMEN medullary thyroid carcinoma. It is usually the first tumor that develops
in these patients and typically occurs in the second or third decade of life.
Tumors are virtually always bilateral and develop in multiple areas of the
middle and upper portions of the thyroid lobe. Detection has historically been
based on elaboration of calcitonin, particularly in response to provocative
testing by the potent secretagogues calcium and pentagastrin. Patients with
medullary carcinoma of the thyroid have striking increases in plasma
calcitonin levels after provocative testing, which allows them to be readily
identified. By employing provocative testing in kindred members at risk for
disease, medullary carcinoma of the thyroid was diagnosed at a preclinical
stage, and a greater percentage of these patients were cured by surgical
therapy. With genetic testing now available, early treatment of medullary
thyroid cancer should be possible for all affected people, to hopefully
enhance the number of people cured of this cancer. The MEN 1, 2a and 2b
syndromes are inherited in an autosomal dominant fashion with complete
penetrance but variable phenotype. Bilateral medullary carcinoma of the
thyroid occurs in every affected patient. 51. While awaiting surgery on a
HMO waiting list, a 50-year-old female with primary hyperparathyroidism is

admitted to the hospital with oliguria, confusion, nausea and vomiting,


muscle weakness and a serum calcium of 13.5 mg/dL. Of the treatment
options below, which one is the most appropriate? a. Administer 1 gm of
hydrocortisone STAT b. Begin an IV EDTA (chelating agent) infusion STAT
c. Administer IV Mithramycin and calcitonin concurrently and proceed to
ICU for cardiac monitoring d. Begin a normal saline infusion at 2X
maintenance volume followed by 1mg/kg furosemide IV e. None of the
above Answer: d Patients with marked hypercalcemia or severe symptoms
should be admitted to the hospital for careful observation and monitoring.
The mainstay of therapy is intravenous hydration, preferably with normal
saline in sufficient quantities to maintain the urine output above 100 mL/h.
These patients are often dehydrated before therapy, and fluid can be
administered intravenously at a rate of 200 mL/h. Caution must be exercised
in older patients who might have marginal cardiac reserve. The diuretic
furosemide also increases excretion of sodium and calcium but should not be
employed until the patient is well hydrated. Saline diuresis is usually
effective when the hypercalcemia results from hyperparathyroidism or from a
benign cause. In contrast, the hypercalcemia of malignancy may produce
severe symptoms associated with extremely high serum calcium levels that
are difficult to control. In this setting, a variety of other measures may be
considered (see Table 57-3). Some of the agents used to treat hypercalcemia
have significant toxicity and require close monitoring. Calcitonin is a fairly
weak hypocalcemic agent, but it acts rapidly and is relatively less toxic.
Glucocorticoids may be particularly efficacious in patients with sarcoidosis
and other granulomatous diseases. Mithramycin has proved useful in patients
with hypercalcemia of malignancy, but it has a substantial cumulative toxicity
(thrombocytopenia, hepatotoxicity, and nephrotoxicity). Intravenous
phosphates and chelating agents have largely been abandoned because of
their severe toxicity. 52. Which of the following conditions are associated
with hypercalcemia? a. Hyperthyroidism b. Thiazide diuretics c. Terminal

ileal resection d. Breast cancer e. All of the above Answer: a, b, d Multiple


causes of hypercalcemia include the following: Malignancy Vitamin A or D
intoxication Thiazide diuretics Hyperthyroidism Milk-alkali syndrome
Sarcoidosis and other granulomatous diseases Familial hypocalciuric
hypercalcemia Immobilization Pagets disease Lithium therapy Addisonian
crisis Idiopathic hypercalcemia of infancy Hyperthyroidism is associated
with increased bone reabsorption. Often, the plasma PTH is low, and a
history of other thyrotoxic symptoms can be elicited. Thiazides may increase
serum calcium to a mild degree, primarily through hemoconcentration. Serum
phosphate may also be depressed. It often takes several weeks for the
hypercalcemia to resolve after the medication is discontinued. Terminal ileal
resection is not associated with hypercalcemia, indeed hypocalcemia may
develop with short gut syndrome. While ileal resection is a predisposing
condition for the development of calcium oxalate kidney stones, this is the
result of alterations in oxalate metabolism. Patients with solid tumors, such as
lung carcinoma, breast carcinoma, squamous cell carcinoma of the head,
neck, esophagus or female genital tract or renal cell cancer account for three
quarters of all cases of humoral hypercalcemia of malignancy. 53. A 40-yearold male undergoes an apparently uneventful total thyroidectomy for
follicular carcinoma of the thyroid. 48 hours later he develops circumoral
numbness, followed by laryngospasm, and then has a generalized seizure. Of
the following, which is the first priority? a. Proceed to OR for exploration of
the operative site b. Administer 25 ml of 10% calcium gluconate
intravenously c. Obtain a serum magnesium measurement and administer
intravenous magnesium chloride STAT d. Obtain a CT scan of the head to
evaluate the possibility of brain metastases Answer: b Postoperative
hypoparathyroidism commonly develops after total thyroidectomy for
malignancy. Most patients undergoing operation on the thyroid experience
some alteration in serum calcium. The hypocalcemia is usually transient and
is not treated unless significant symptoms develop. The plasma calcium

usually reaches its nadir at 48 to 72 hours after surgery and then slowly
returns to normal over 2 to 3 days. For these reasons, careful postoperative
monitoring of serum calcium levels is essential. For acute symptomatic
hypocalcemia, calcium should be administered intravenously. Usually 20 to
30 mL of 10% calcium gluconate is infused over a 15 to 20 minute period
and then 50 to 100 mL are administered over the next 12 hours in adults.
Calcium gluconate is less irritating to the veins than calcium chloride, and the
calcium release is slower without the risk of overcorrection. Serum
magnesium should always be measured and hypomagnesemia should be
corrected if present. This is not the first priority for this patient. Symptoms
should never be allowed to progress to the point demonstrated in this patient.
Postoperative routines include careful monitoring of the serum calcium until
stable. There is no evidence that this patient has a postoperative surgical
problem such as a cervical hematoma. Laryngospasm and seizures are
classical signs of hypocalcemia. Lastly, the likelihood of metastatic brain
disease is very low in this circumstance. 54. On routine life insurance
screening, an asymptomatic 45-year-old male is found to have a serum
calcium level of 12.5 mg/dL. Serum is obtained for immunoreactive
parathyroid hormone level and this is 400 mEq/mL (normal range <
64mEq/mL). These findings are most consistent with a diagnosis of which of
the following? a. Primary hyperparathyroidism b. Secondary
hyperparathyroidism c. Tertiary hyperparathyroidism d. Ectopic
hyperparathyroidism Answer: a Primary hyperparathyroidism develops
spontaneously without apparent cause, but possibly in response to exogenous
stimuli. When the normal control of serum calcium is disturbed and there is
increased autonomous production of PTH, the state is referred to as primary
hyperparathyroidism. In contrast, secondary hyperparathyroidism occurs
when there is a defect in mineral homeostasis that leads to a compensatory
increase in parathyroid function. This occurs most commonly in response to
renal disease, but may also develop as a consequence of the hypocalcemia

associated with some diseases of the gastrointestinal tract, bone or other


endocrine organs. Occasionally, with prolonged secondary stimulation, the
hyperfunctioning glands are no longer physiologically responsive to an
increased ionized calcium. This rare, relatively autonomous state is referred
to as tertiary hyperparathyroidism and develops most commonly after renal
transplantation when the defect in calcium homeostatis is corrected. The
numerical values for calcium and PTH here are consistent with primary
hyperparathyroidism. Relation between serum immunoreactive parathyroid
hormone (iPTH) and serum calcium in patients with hypoparathyroidism,
pseudohypoparathyroidism, ectopic hyperparathyroidism, and primary,
secondary, and tertiary hyperparathyroidism. GP1M, guinea pig antiserum
1M. (After Clark OH, Way LW. Thyroid and parathyroid. In: Current surgical
diagnosis and treatment, ed 8. Norwalk, CT, Appleton & Lange, 1989:249)
55. A 50-year-old female is referred for treatment of a serum calcium of 11.5
mg/dL one year after resection of a right superior parathyroid adenoma. The
right and left inferior glands were normal at the initial neck exploration. The
left superior gland was not visualized. Which of the related statements below
are true? a. Recurrent postoperative hypercalcemia occurs in approximately
20% of patients with this clinical scenario b. Localization studies via
selective angiography are successful in 50% to 80% of these patients c. The
risk of permanent hypoparathyroidism is approximately 10% to 20%
following reexploration in this setting d. Surgical reexploration by an
experienced endocrine surgeon has a success rate of less than 50% in this
circumstance Answer: b, c Persistent hyperparathyroidism occurs in less than
5% of patients after neck exploration for primary hyperparathyroidism by an
experienced surgeon. Most commonly, it is the result of a single diseased
gland still remaining in the neck or in the mediastinum. Recurrent disease
develops after an interval of normocalcemia and may be the result of
regrowth of diseased tissue, implantation from a tumor broken at the initial
procedure, or even recurrent parathyroid carcinoma. Documentation of a

correct initial diagnosis and review of the original operative and pathology
reports are essential. It is generally agreed that localization studies do have a
place in the management of recurrent disease. Noninvasive methods are
employed first, and if these are unsuccessful in identifying the diseased
gland, selective angiography and venous sampling for PTH are employed.
Selective angiography appears to be the most accurate technique,
successfully localizing 50% to 80% of parathyroid glands that cannot be
detected by any other modality. Surgical reexploration can be difficult.
Generally the neck is explored first after which the mediastinum is examined
if this is unproductive. Surgical reexploration is successful in experienced
hands in 60% to 80% of cases. There is, however, an increased incidence of
complications. Unilateral recurrent nerve injury occurs in 5% to 10% of
patients and permanent hypoparathyroidism occurs in 10% to 20% of patients
postoperatively. 56. Which of the following statements regarding calcium and
phosphate metabolism are true? a. Parathyroid hormone excess produces a
net increase in daily urinary calcium excretion b. Calcitonin is essential for
the normal control of calcium metabolism in adult humans c. Parathyroid
hormone is the single most important regulator of calcium and phosphate
metabolism in humans d. 1,25 dihydroxyvitamin D3 (calcitriol) is the active
form of vitamin D in humans Answer: a, c, d The primary hormonal
regulators of calcium and phosphate metabolism are parathyroid hormone
(PTH), vitamin D and calcitonin. Parathyroid hormone appears to be the
single most important hormonal regulator of calcium and phosphate
metabolism in humans. It has direct effects on the skeleton and kidney and
indirect effects on the intestine, mediated through vitamin D. Sustained
elevations of PTH stimulate osteoclasts and inhibit osteoblasts leading to
absorption of calcium from bone. In the kidney, PTH produces an increase in
reabsorption at any given concentration of extracellular fluid calcium,
although excess secretion, because of the hypercalcemia, results in a net
increase in daily urinary calcium excretion. Vitamin D3, or cholecalciferol, is

produced normally by the action of sunlight on 7-dehydrocholesterol in the


skin. It is then hydroxylated in the liver (25 position) and kidney (1 position)
to form the active 1,25 dihydroxyvitamin D3 (calcitriol). This is the active
form of Vitamin D in humans. Calcitonin is a 32-amino acid protein produced
by the parafollicular C cells of the thyroid. Total thyroidectomy, with removal
of all the C cells, is well tolerated, and it has been concluded that calcitonin is
not essential for the normal control of calcium metabolism in adult humans.
Calcitonin does inhibit bone resorption and can produce hypocalcemia in
experimental animals. It also increases urinary calcium and phosphate
excretion. 57. Multiple Endocrine Neoplasia (MEN) 2b is characterized by
which of the following findings? a. Medullary carcinoma of the thyroid,
pheochromocytoma, mucosal neuromas, and a distinctive marfanoid habitus
b. Parathyroid hyperplasia, pancreatic islet cell tumors, and pituitary
adenomas c. Medullary carcinoma of the thyroid, pheochromocytoma, and
parathyroid hyperplasia d. Parathyroid carcinoma, pheochromocytoma and
chronic pancreatitis Answer: a The multiple endocrine neoplasias are familial
disorders typically characterized by predisposition to the development of
tumors of multiple endocrine organs. These disorders are all inherited in an
autosomal dominant fashion, and the tumors tend to be multicentric. The
tumors may be benign or malignant and may occur metachronously or
synchronously. MEN 1 is characterized by the concurrence of parathyroid
hyperplasia, pancreatic islet cell tumors, and pituitary adenomas. MEN 2a
consists of medullary carcinoma of the thyroid, pheochromocytoma, and
parathyroid hyperplasia. MEN 2b includes medullary carcinoma of the
thyroid, pheochromocytoma, mucosal neuromas, and a distinctive marfanoid
habitus. Together, these syndromes encompass much of the spectrum of
endocrine neoplasia. 58. Which of the following signs/symptoms are
pathognomonic of hyperparathyroidism? a. Pathologic fractures of the
metacarpals b. Calcium oxalate nephrolithiasis c. Hypercalcemia causing
mental status changes d. Atrophy of Type II muscle fibers e. Osteitis fibrosa

cystica Answer: e Manifestations of hyperparathyroidism are protean, but


generally nonspecific and may be difficult to elicit in the history. The earliest
complaints are often vague and include muscle weakness, anorexia, nausea,
constipation, polyuria, and polydipsia. These nonspecific symptoms may or
may not cause the patient to seek medical attention. Symptomatic patients
generally have evidence of chronic disease involving the kidney or the
skeleton. Renal complications develop because the hypercalcemia leads to
increased urinary calcium excretion and PTH increases the excretion of
phosphate. These events predispose to stone formation. Nephrolithiasis
develops in about 30% of patients. Nephrocalcinosis occurs in 5% to 10% of
patients. These are both nonspecific for hyperparathyroidism. Parathyroid
bone disease in its most classic and severe form, osteitis fibrosa cystica, is
seldom seen. However, 5% to 15% of patients present with significant
symptoms of skeletal disease. Only the skeletal disease is pathognomonic.
Bone changes include subperiosteal resorption of bone on the radial aspect of
the middle phalanx of the second or third finger. Tufting of the distal
phalanges, bone cysts of the skull and long bones, brown tumors (i.e.,
localized proliferations of osteoclasts), and diffuse demineralization or
granularity are seen as well. Atrophy of Type II muscle fibers, consistent with
a neuropathic and not a myopathic cause, has been demonstrated in patients
with hyperthyroidism, but this too is nonspecific. 59. The causes of Cushing
syndrome may include which of the following? a. Posterior pituitary
adenoma b. Adrenal hyperplasia c. Small cell lung carcinoma d.
Pheochromocytoma e. Adrenal carcinoma Answer: b, c, e The varied causes
of cortisol excess produce clinical features that are collectively called
Cushing syndrome. These include exogenous steroid administration, Cushing
disease (excessive ACTH production by the anterior pituitary gland, usually
from an adenoma), ectopic ACTH production (small cell lung carcinoma),
adrenal adenoma or carcinoma, micronodular pigmented hyperplasia,
macronodular hyperplasia, and steroid-dependent adrenal hyperplasia.

Pheochromocytoma is characterized by catecholamine rather than cortisol


excess as it arises from the adrenal medulla rather than the adrenal cortex. 60.
Which of the following are normal systemic effects of glucocorticoids? a.
Enhanced proteolysis b. Increased gluconeogenesis c. Diminished lipolysis d.
Decreased rate of intestinal epithelial replication Answer: a, b, d The many
systemic effects of glucocorticoids are related to regulation of intermediary
metabolism. In this regard, perhaps the most important action is the effect of
steroids on protein breakdown. A direct proteolytic effect of steroids is
suggested by several lines of evidence. Glucocorticoids enhance
gluconeogenesis by both a direct effect on gluconeogenic hepatic enzymes
and also by provision of substrate for gluconeogenesis by proteolysis.
Glucocorticoid influence leads to the accentuation of lipolysis. The truncal
obesity seen in steroid excess is related to the predominance of the lipogenic
effect of insulin on truncal adipocytes over the lipolytic effect of
glucocorticoids. The opposite relation may hold for the receptors in fat of the
extremities and would explain the comparatively scant fat in these areas with
steroid excess. The most notable effect of glucocorticoids in the intestinal
tract is a decrease in the rate of mucosal cell replication. In addition,
decreased mucosal and pancreatic prostaglandin synthesis occur. This may
have important implications for the cytoprotective mechanisms in the
stomach. 61. A 10-year-old child presents with hypertension, tachycardia,
nervousness and sweating. The best initial diagnostic evaluation is which of
the following? a. Radioimmunoassays for norepinephrine and epinephrine in
serum b. Magnetic resonance imaging of the adrenal gland c. MIBG (131IMethaiodobenzylguanidine) scintigraphy d. Measurement of catecholamines
and their degradation products in a 24-hour urine specimen Answer: d The
first diagnostic step in determining the functional state of an adrenal gland or
lesion is to screen the urine or plasma for secretory products. Once
hypersecretion is demonstrated, the specific type of pathology producing a
syndrome must be determined with the aid of functional tests and relevant

scanning and imaging. The most efficient and sensitive means of screening in
a patient suspected of having a pheochromocytoma, (as indicated here) is
measurement of the catecholamines or metabolic products thereof in the
urine. Although 24-hour samples can smooth out the possible episodic
variations in catecholamine secretion, shorter sampling periods can be useful,
especially if corrected for creatinine excretion. Timing of the collection is
critical in patients who have only episodic hypertension. Urine collection
should be started immediately after a suspected attack of hypertension.
Fluctuations in plasma catecholamine concentrations are much greater than
those in urinary excretion, even in normal subjects. Plasma determinations
are quite sensitive and specific with radioimmunoassays and HPLC
determination of plasma catecholamines, but the specificity can be low
because of the overlap of normal spikes in catecholamine concentrations with
concentrations produced by minimally secreting pheochromocytomas. MRI
and MIBG imaging studies are both potentially useful after catecholamine
excess is confirmed. MRI demonstrates anatomy quite well. MIBG
scintigraphy is particularly useful in looking for nonadrenal and bilateral
pheochromocytomas. 62. A 25-year-old male has been taking 40 mg of
prednisone PO qod for ulcerative colitis for 5 years. He undergoes an
uneventful colectomy with endorectal pull-through and an ileoanal
anastomosis. Which of the following statements regarding steriod
management are correct? a. On the day of surgery he should receive 100 mg
hydrocortisone IV q6h b. The postoperative steroid dose should be halved q
12 hours to reduce the risk of infectious complications and improve would
healing c. Prophylactic treatment with a somatostatin analogue will reduce
the risk of postoperative pancreatitis d. Exogenous steroid replacement can be
stopped after 3 months Answer: a The postoperative course of management
for a patient with pituitary adrenal suppression from exogenous steroids
involves tapering the exogenous steroid doses to maintenance levels after
high dose replacement at the time of operation. One simple regimen involves

administering 100 mg of hydrocortisone intravenously every 6 hours during


the first 48 hours. Some prefer alternating doses of IM cortisone acetate in
the event that intravenous access is lost. Provided that no intervening
complications arise, the doses can be halved every 48 to 72 hours. In patients
who have been exposed preoperatively to glucocorticoid excess, the
maintenance dose may be as high as 100 mg/d for several months. Both high
doses and normal maintenance of 35 to 55 mg/d can be given in the form of
oral cortisone acetate as long as reliable alimentation and absorption have
been achieved. The pituitary-adrenal axis remains suppressed for 6 to 12
months after operation. Complications in the postoperative period include
wound infection, pancreatitis, and thromboembolism. There is no data to
suggest that the risk of postoperative pancreatitis can be diminished with
somatostatin analogue therapy. 63. Imaging of the adrenal gland is best
achieved with which of the following techniques? a. Ultrasound b. Computed
tomography (CT) c. Arteriography d. Scintigraphy with 131I-6 b-iodomethyl19-norcholesterol (NP-59) e. Scintigraphy with 131Imethaiodobenzylguanidine (MIBG) Answer: b Although ultrasonography is
the least expensive of the imaging procedures, its value is limited by the
relative inaccessibility of the adrenal gland and by the small size of some
adrenal lesions. CT is the technique most commonly used to examine patients
in whom adrenal abnormalities are suspected. CT reliably detects adrenal
tumors greater than 1 cm in diameter. The sensitivity of CT for tumors that
are 1 cm in diameter is about 80% and reaches 100% for tumors that are 3 to
4 cm. Although CT is noninvasive and reasonably sensitive, it is nonspecific.
It does not distinguish functioning from nonfunctioning tumors, nor benign
from malignant tumors with any degree of reliability. MRI has developed a
certain usefulness even after retrenchment from early optimistic predictions.
MRI is more expensive and requires greater patient cooperation than CT, but
it has greater versatility because of the use of TI-and T2-weighted images. In
some cases, the T2-weighted images are capable of providing a differential

diagnosis and may distinguish such entities as metastatic or primary


carcinoma and pheochromocytoma from adenomas, lipomas, myelolipomas
and cysts. In a sense, MRI is complementary to CT in that the latter can better
detect the lesion while the former can distinguish one type of lesion from the
other. In addition, MRI is probably better than CT for distinguishing
anatomic relationships and the extent of involvement of the surrounding
tissues. Two radiopharmaceuticals have proved useful in imaging the adrenal
gland. Adrenocortical lesions can be imaged by NP-59 which is taken up as
cholesterol in the adrenocortical steroidogenic pathway. The other agent is
MIBG, a norepinephrine analogue. It indicates norepinephrine accumulation
in storage vesicles and can detect sympathoadrenal tumors at any site in the
body. NP-59 can localize the adrenal cortex and any functioning tumors. It
can distinguish adrenocortical hyperplasia from functioning adenomas or
carcinomas. MIBG is a useful agent in localizing pheochromocytomas
throughout the body, especially when the tumors are multiple, extraadrenal,
recurrent, or metastatic. Arteriography, venography and selective venous
sampling have become less popular as experience with other imaging
techniques has become greater. Disadvantages inherent in invasive
procedures using intravascular contrast agents are obvious. Arteriography is
specifically dangerous in the study of patients with pheochromocytomas. 64.
A 45-year-old female is found to have a 2 cm solid nodule in her right adrenal
gland at the time of an abdominal CT scan following an auto accident. With
regard to the adrenal lesion, she is asymptomatic and it is found to be
nonfunctional on evaluation. You would recommend which of the following?
a. Extraperitoneal right adrenalectomy through either a flank of posterior
approach b. Suppression with 5 mg prednisone PO qod c. Followup CT scan
in 1 to 3 months d. Excisional biopsy via laparaoscopic approach Answer: c
Indication for operation in the patient with a unilateral functioning adrenal
tumor is clear. In the patient with a nonfunctioning adrenal tumor, the need
for surgery is related to the size of the tumor and its rate of growth. There is

consensus that a tumor larger than 6 cm should be removed. Some


recommend that the acceptable size limit be 3 cm, especially when MRI
suggests carcinoma or when the functional studies suggest activity. When
nonoperative therapy is elected, the patient should receive an adrenal scan 1,
3, and 6 months after the initial scan and yearly thereafter to assess the
growth of the lesion. If the tumor has grown, surgical removal is indicated.
65. A 20-year-old male with a 10 cm left adrenal mass is found to have 10 mg
of norepinephrine in a 24-hour urine collection and a plasma 18
hydroxycorticosterone level of 50 mg/dL. Initial reoperative preparation
should include which of the following? a. Treatment with spironolactone b.
Intravenous potassium-loading to prevent intraoperative hypokalemia c.
Treatment with phenoxybenzamine d. Treatment with labetalol Answer: c
This patient has a pheochromocytoma. The most efficient and sensitive
means of screening for pheochromocytoma is measurement of the
catecholamines, or metabolic products thereof, in the urine. The
catecholamines, norepinephrine and epinephrine, are excreted in amounts
under 100 g/d in the normal person. Because of some overlap in values,
specificity can be improved by using a normal range of up to 250 mg/d. The
measurement of plasma 18-hydroxycorticosterone level is helpful in
evaluating patients with hyperaldosteronism, as it is an intermediate product
in its synthesis. The 18-hydroxycorticosterone levels are above 100 mg/dL in
virtually all patients with aldosterone producing adenomas. The plasma value
for this patient is normal. Perioperative treatment with either spironolactone
and potassium replacement is appropriate for patients with
hyperaldosteronism, but not pheochromocytoma. Nonoperative treatment of
pheochromocytoma is generally unsatisfactory and entails pharmacologic
blockade of the effects of catecholamines. Phenoxybenzamine and prazosin
are two preferred agents that block the a-adrenergic effects of the
catecholamines preoperatively with pheochromocytoma. The use of badrenergic blockers, such as labetalol, may be required in those patients with

obvious b-adrenergic effects, such as resting pulse rates above 100 beats/min.
Because of the potential for wide swings in blood pressure and other effects
of chronic catecholamine secretion, careful preoperative preparation is
required in patients with pheochromocytoma. It is customary to institute aadrenergic blockade 2 to 3 weeks before anticipated surgery. This has
beneficial effects of controlling blood pressure and allowing restoration of a
decreased blood volume. It is the consensus that preoperative preparation in
the manner makes the intraoperative management of the patient much more
safe. In patients who require b-adrenergic blockade, it is essential to first
establish good a-adrenergic blockade. These patients are prone to cardiac
failure induced by b-adrenergic blockade because of the cardiomyopathy that
may preexist. b-Adrenergic blockade in the cardiomyopathic patient with
failure to first reduce the afterload by a-adrenergic blockade, can precipitate
cardiac failure. 66. Which of the following diagnostic tests will differentiate
between the pituitary and adrenal gland as the cause of hypercortisolism? a.
High dose dexamethasone suppression test b. CRH stimulation test c. Low
dose dexamethasone test d. AM and PM plasma cortisol determination
Answer: a, b Diurnal variation of plasma cortisol levels is lost in
hypercortisolism, caused by both adrenal tumors and pituitary lesions.
Dexamethasone, by negative feedback, suppresses the hypothalamic pituitary
secretion of ACTH and effects the consequent lowering of both plasma
cortisol and urinary 17-hydroxycorticosteroid excretion. Administration of a
single dose of 2 mg of dexamethasone suppresses plasma cortisol and urinary
17-hydroxycorticosteroids by at least half when compared to control values
from untreated normal patients (low dose suppression test). In Cushing
disease, with a setpoint of ACTH secretion higher than normal, low-dose
dexamethasone is insufficient to suppress ACTH. High-dose dexamethasone
suppression is achieved by 2 mg of dexamethasone administered every 6
hours for 24 hours. A normal response is to lower the 17hydroxycorticosteroid excretion by more than one half. In the case of

Cushing disease, the hypothalamic steroid receptors that allow negative


feedback are intact but at a higher setpoint. In this case, 17hydroxycorticosteroid secretion does decrease significantly after high-dose
dexamethasone administration. Adrenal tumors, other causes of ectopic
production of ACTH, and most cases of nodular hyperplasia do not respond
to dexamethasone suppression with a decrease in steroid secretion. With an
adrenal tumor, pituitary ACTH is already suppressed; therefore,
dexamethasone cannot suppress it further. With ectopic ACTH secretion, the
tissue producing ACTH has no receptors for steroids and thus negative
feedback cannot be achieved. Therefore, high dose dexamethasone
suppression will differentiate hypercortisolism of pituitary and adrenal
origins. Potentially the most helpful new test for this purpose uses the
administration of recombinant CRH to release ACTH and consequently to
stimulate cortisol secretion. CRH (1 mg/kg) is administered intravenously,
and serial blood samples obtained for 3 hours after administration. The
normal pituitary adrenal axis responds by a moderate increase in ACTH and
cortisol. With Cushing disease, the ACTH and cortisol rise are accentuated.
With adrenal autonomous production of cortisol and with ectopic ACTH
production, there is virtually no response to CRH. 67. Which of the following
statements regarding androgens and estrogens in the fetus are true? a. The
development of normal female external genitalia requires estrogen production
by the ovary b. A female with congenital adrenal hyperplasia is likely to be
masculinized in appearance c. The development of normal male external
genitalia requires adrenal androgen production d. A male with congenital
adrenal hyperplasia is likely to be feminized in appearance Answer: b, c
Adrenal androgens in the fetus stimulate wolffian duct development and
elongate the genital tubercle. They promote midline migration of the labial
folds and fusion of these folds to form the scrotum. To complete the male
transformation, the urethral opening migrates to the tip of the phallus. All of
these events are androgen-dependent. Since the normal female fetus does not

secrete androgens, the genital tubercle, labial folds, and urethral opening all
remain in the female position in this circumstance. Thus the female
phenotype is associated with the absence of fetal sex hormone production.
Excess androgen in the female fetus causes neonatal virilization, as is seen
with congenital adrenal hyperplasia. The male infant with congenital adrenal
hyperplasia is likely to have a normal appearance of the external genitalia as
a neonate. Precocious puberty will develop over a period of years in this
latter circumstance. 68. Which of the following statements regarding the
physiology of the adrenal gland are true? a. Release of CRH is regulated
principally by negative feedback by ACTH b. Plasma 17-ketosteroid levels
reflect the degree of adrenal cortisol production c. Renin undergoes
enzymatic cleavage in the lung to angiotensin I d. The plasma half-life of
ACTH is relatively long (> 24 hrs)
e. None of the above
Answer: e
The proximate stimulator of cortisol production is the peptide hormone,
andrenocorticotropic hormone (ACTH). It originates from the anterior
pituitary gland and is regulated by corticotropin-releasing hormone (CRH).
The regulation of CRH is controlled by various neural influences. These
include intrinsic central nervous system influences and a negative feedback
inhibition by cortisol. Although there is some evidence of a short-loop
feedback of ACTH on CRH, both slow and fast feedback by cortisol on the
pituitary release mechanism are the primary sources of clinically relevant
CRH regulation.
The steroidogenic pathway involves the conversion of cholesterol to
pregnenolone, progesterone, 17-hydroxyprogesterone, and then either to the
adrenal androgens or cortisol via several intermediates. The 17-ketosteroids
reflect adrenal androgen synthesis while the 17-hydroxysteroids reflect
cortisol synthesis.

Renin is produced predominantly in the juxtaglomerular apparatus of the


kidney where it acts locally and is released into the systemic circulation.
Renin cleaves angiotensin I, a decapeptide derived from the liver which
serves as renin substrate. Angiotensin I undergoes enzymatic cleavage in the
lung to angiotensin II, which is the biologically active form of the peptide.
The plasma half-life of ACTH is short (measured in minutes) with a rapid
onset of action. This is in contrast to a longer plasma half-life and a slower
onset of action for cortisol itself.

69. The approximate 5-year survival rate for adrenocortical carcinoma is


which of the following?
a. 0%
b. 20%25%
c. 50%60%
d. Nearly 100%
Answer: b
The prognosis for adrenocortical carcinoma is not good. The overall 5-year
survival rate is 20% to 25% for these malignancies. When there is localized
disease at the time of surgery, the 5-year survival may be higher, in the 40%
to 50% range. The true prognosis in childhood is not clear, but the data
suggest a 2-year survival rate of about 20%.

70. Which of the following statements is true with respect to


pheochromocytoma?
a. Pheochromocytoma associated with MEN IIa is usually unilateral and

rarely malignant; therefore, unilateral exploration through a posterior flank


incision is usually sufficient
b. Clonidine fails to suppress basal plasma-catecholamine levels in patients
with pheochromocytoma
c. NP-59 (131I-6 b-iodomethyl-19-norcholesterol) is taken up as cholesterol
by the adrenal medulla
d. The ratio of plasma 3,4-dihydroxyphenoglycol (DHPG) to norepinephrine
is generally elevated in patients with pheochromocytoma compared to
patients with essential hypertension
Answer: b, d
Pheochromocytoma associated with the MEN IIa syndrome is more often
bilateral and more often malignant, therefore, abdominal exploration through
an anterior approach is indicated. The ability to measure catecholamines in
the plasma has made possible the clonidine suppression test. In patients
without pheochromocytoma, clonidine suppresses high basal plasma
concentrations into the normal range, whereas concentrations in patients with
pheochromocytoma are not suppressed. Another use of plasma catecholamine
measurements is in examining the ratio of 3,4-dihydroxyphenoglycol
(DHPG) to norepinephrine in plasma. DHPG is released from the chromaffin
cells and adrenergic neurons to a much greater extent than norepinephrine in
pheochromocytoma patients compared with patients who have essential
hypertension, i.e. the ratio of DHPG to norepinephrine is higher in patients
with pheochromocytomas.
NP-59 (131I-6 b-iodomethyl-19-norcholesterol) is taken up as cholesterol by
the adrenal cortex and is incorporated in the adrenocortical steroidogenic
pathway. This is a useful agent for imaging adrenocorticol lesions. 131Imethaiodobenzylguanidine (MIBG) is a norepinephrine analogue that is
useful in localizing pheochromocytomas throughout the body, especially
when the tumors are multiple, extraadrenal, recurrent, or metastatic.

71. Which of the following statements regarding aldosterone are true?


a. Its secretion is directly related to the serum potassium concentration
b. Angiotensin II is a more potent regulatory factor than ACTH
c. Primary hyperaldosteronism is characterized by hyperkalemia
d. Secondary hyperaldosteronism occurs with renal artery stenosis
Answer: a, b, d
Aldosterone secretion is controlled by changes in the afferent arteriolar
pressure in the renal cortex as well as by changes in sodium content in the
renal tubule. These changes are sensed by the juxtaglomerular apparatus and
by the macula densa and act through the renin angiotensin system. At least
two other factors influence aldosterone secretion. Aldosterone secretion is
directly related to the serum potassium concentration. An increase in serum
potassium directly stimulates aldosterone production, whereas a decrease in
serum potassium has the opposite effect. Because of its early point of action
in the steroidogenic pathway, ACTH also increases secretion of aldosterone,
although it is much less potent in this regard than in its stimulation of
cortisol. The stimulatory effects of potassium and ACTH on aldosterone
secretion can be overcome by angiotensin II stimulation.
Primary hyperaldosteronism is characterized by mineralocorticoid
hypersecretion which promotes a positive sodium balance and hypokalemia.
About 80% of patients with primary hyperaldosteronism have serum
potassium levels of 3.5 mEq/L or less. Causes of secondary
hyperaldosteronism are related to increased renin secretion. These include
renal artery stenosis, congestive heart failure and renal salt-wasting.

72. A term neonate is noted to have ambiguous female genitalia. This infant is
at risk for which of the following potentially life-threatening problems?
a. Cardiomyopathy with congestive heart failure
b. Sodium wasting nephropathy with hypovolemia
c. Respiratory failure from surfactant deficiency
d. Spontaneous hemorrhage from thrombocytopenia
e. Pulmonary embolus from a hypercoaguable state
Answer: b
Enzymatic defects in the steroidogenic pathway produce a syndrome known
as congenital adrenal hyperplasia. This syndrome presents predominantly in
the neonatal period with sexual ambiguity. These enzymatic defects result in
a lowered cortisol secretion. The specific enzyme defects present determine
the clinical form of the syndromes. These include a 21-hydroxylase
deficiency, an 11b-hydroxylase deficiency, and a 17-hydroxylase-deficiency.
The 21-hydroxylase deficiency and the 11b-hydroxylase deficiency result in
excess androgen production in utero and result in masculinization with
ambiguous genitalia in the female newborn. Masculinizing effects in the male
may not be detected until precocious puberty becomes obvious. About 40%
of patients with 21-hydroxylase deficiency, the most common form, have
salt-wasting or sodium loss by urine. Hypovolemic shock can result.
Cardiomyopathy, respiratory failure, thrombocytopenia and pulmonary
emboli are not associated with this syndrome.

73. Which of the following adrenal lesions can be treated definitively by


medical means?
a. Benign functional adrenocortical adenoma

b. Adrenocortical carcinoma
c. Congenital adrenal hyperplasia
d. Cushing disease
e. Pheochromocytoma
Answer: c
The treatment of adrenal tumors is primarily surgical removal. Although
pharmaceutical agents are useful in preparing the patient for surgery or in
palliating the patient with recurrent adrenal carcinoma, no agents render
definitive therapy for adrenal tumors. Congenital adrenal hyperplasia stands
alone among the primary, hyperfunctioning adrenal syndromes that are
amenable to medical therapy for definitive treatment. Functioning benign
lesions of the adrenal cortex that are not ACTH dependent, such as adenomas
or macronodular hyperplasia, respond to metyrapone and aminoglutethimide,
which are inhibitors of enzymes in the adrenal steroidogenic pathway. Both
agents can effect a decrease in the production of cortisol when there is no
increase in ACTH secondary to feedback stimulation. These drugs are not
satisfactory long-term agents because of their high incidence of drug
reactions, patient noncompliance, and continued growth of the lesions. They
may be useful in patients whose surgery must be delayed. Although
malignant, functioning adrenocortical lesions should be debulked whenever
possible. Several chemotherapy agents offer adjunct therapy. The most
noteworthy is mitotane (o,p,-DDD). This is a cytolytic agent that has a 30%
to 70% response rate in terms of decreasing steroid output. Unfortunately,
patient survival is not affected. As mentioned, nonoperative treatment with
cortisone acetate and possibly fludrocortisone is definitive therapy for
congenital adrenal hyperplasia. Cushing disease is best treated by
transsphenoidal resection of the pituitary adenoma. Pheochromoctyoma
requires definitive surgical resection although the preoperative
pharmacologic preparation with catecholamine blockade is required.

74. Which of the following statements regarding the pituitary gland are true?
a. ADH is a product of the neurohypophysis
b. The preferred surgical approach to the pituitary gland is via the sphenoid
sinus
c. Growth hormone, ACTH, LH, FSH and serotonin are products of the
adenohypophysis
d. The adenohypophysis is regulated by neurotransmitters released by the
supraoptic hypophyseal tract
Answer: a, b
The anterior pituitary gland is the adenohypophysis which constitutes 80% of
the gland. The posterior pituitary, the neurohypophysis, constitutes the
remainder and should be considered virtually an extension of the
hypothalamus of the brain. The pituitary resides within bony confines of the
sella turcica (Turkish saddle) and is bordered laterally by the cavernous
sinuses (venous), inferiorly and anteriorly by the sphenoid sinus (air),
posteriorly by the dorsum sella and superiorly by the membranous
diaphragma sella. The cavernous sinuses each contain the siphon region of
the internal carotid artery and portions of the cranial nerves III, IV, V and VI
all within the venous plexus. The optic chiasm lies immediately above the
diaphragma sella. Directly below the anterior and inferior portions of the
sella is the aerated sphenoid sinus. This is sufficiently large in 97% of the
patients to allow a transnasal, transsphenoidal surgical approach to the
pituitary.
The adenohypophysis is regulated by a portal venous system between the
median eminence of the hypothalamus and the adenohypophysis itself. This
system involves a transport of (1) thyrotropin-releasing hormone (TRH), to
stimulate the secretion of the thyroid-stimulating hormone (TSH); (2)

corticotropin-releasing hormone, to stimulate adrenocorticotropic hormone


(ACTH); (3) growth hormone-releasing hormone, to stimulate secretion of
growth hormone (GH); (4) gonadotropin-releasing hormone, to stimulate
luteinizing hormone (LH) and follicle-stimulating hormone (FSH); and (5)
prolactin-inhibitory factor (dopamine), to inhibit prolactin. The
neurohypophysis is regulated by means of direct transport of hormones
through nerve fibers from the supraoptic and paraventricular nuclei in the
hypothalamus. The neurohypophysis is a virtual extension of the
hypothalamus. Products of the neurohypophysis are antidiuretic hormone
(ADH; vasopressin) and oxytocin. The pituitary gland is not known to release
serotonin.

75. Which of the following statement(s) is/are true with respect to growth
hormone secreting pituitary adenomas?
a. Fewer than 50% of patients will have growth hormone levels over 10
ng/mL
b. Oral glucose administration suppresses growth hormone levels in patients
with acromegaly
c. Over 80% of growth hormone-secreting microadenomas can be cured with
transphenoidal resection
d. Preoperative treatment of macroadenomas with a somatostatin analogue
may improve postoperative remission rates
Answer: c, d
The endocrine diagnosis of acromegaly rests largely on serum growth
hormone (GH) levels, because 90% of patients will have levels over
10ng/mL. When acromegaly is apparent but consistently elevated growth
hormone levels are not obtained, the glucose suppression test is the most

useful diagnostic procedure. In normal patients, 1 to 2 hours after the oral


administration of 100 g of glucose, the growth hormone level falls well below
5 ng/mL. This suppression is not seen with GH-secreting adenomas, and
often a paradoxical rise in GH is observed.
The goals of treatment are to lower the circulating growth hormone or
somatomedin C levels to within a normal range and to reduce the size of the
mass lesion causing compression-related symptoms. When a microadenoma
is removed transsphenoidally, endocrine remission may be expected in 80%
to 88% of cases. When a macroadenoma is resected, postoperative remission
is reported in 30% to 68% of cases. The rate of remission is inversely related
to preoperative GH levels and tumor size. Preoperative treatment of
macroadenomas with a somatostatin analogue may improve postoperative
remission rates.

76. A 30-year-old woman presents with amenorrhea, headache and


bitemporal hemianopsia. Appropriate diagnostic tests include:
a. Cerebral angiography
b. Serum prolactin levels
c. Magnetic resonance imaging of the brain
d. Abdominal and pelvic CT scan
Answer: b, c
Patients with pituitary lesions present symptoms and signs related to a mass
effect on the pituitary and its surrounding structures, to hypersecretion of the
hormones by the lesion itself, or to a combination of both. As mass lesions in
the pituitary enlarge, they encounter the various contents of the cavernous
sinuses, including the third, fourth, sixth and first two divisions of the fifth
cranial nerves, as well as the internal carotid artery. The growth of a tumor in

the relatively unrestricted upward direction is much more common and often
results in compression of the optic chiasm with the resultant loss of vision,
typically a bitemporal hemianopsia. Prolactin-secreting pituitary adenomas
often present with endocrine symptoms including amenorrhea and
galactorrhea in women. In men, the loss of libido, infertility and visual loss
are typical. Magnetic resonance imaging (MRI) has evolved as the first
choice for diagnostic imaging and is often the only tool needed to reach a
therapeutic decision with regard to pituitary adenomas. With intravenous
infusion of a paramagnetic substance such as gadolinium, MRI demonstrates
intrasellar tumors as small as 5 mm. In addition, the extent of suprasellar and
sphenoid sinus extension, as well as lateral extension into the cavernous
sinuses, is demonstrable. Cysts and hemorrhage can be differentiated, as can
blood flowing within an aneurysm. CT scanning has a place in pituitary
imaging if MRI scanning is unavailable. Plain skull X-rays are not needed
generally. Cerebral angiography is performed only if an aneurysm is
suspected or if a lesion is so large that occlusion or compression of the
internal carotid artery is in question. For this patient, the symptoms clearly
point to a central nervous system, pituitary etiology rather than abdominal
end organ failure with regard to the amenorrhea.

77. Which of the following condition(s) is/are associated with


hyperprolactinemia?
a. Chronic renal failure
b. Exogenous estrogen administration
c. Diabetes mellitus
d. Cirrhosis
Answer: a, b, d

Elevated serum prolactin levels do not always indicate the presence of a


pituitary tumor. Important alternative causes are chronic renal failure,
hypothyroidism, various drugs including phenothiazines, tricyclic
antidepressants, exogenous estrogen, opiates, reserpine, verapamil and others.
In addition, hepatic disease, pregnancy and a variety of pituitary and
hypothalamic lesions cause hyperprolactinemia. If the prolactin level is over
150 ng/ml, a pituitary tumor is almost invariably the cause, but often
microadenomas produce prolactin levels of less than 100 ng/ml. The size of
pituitary tumors has been shown to relate to the degree of prolactin elevation,
which may reach into the thousands of nanograms per milliliter. There are no
reliable provocative tests to differentiate prolactinomas from other causes of
hyperprolactinemias, so the diagnosis relies on ruling out other causes and
imaging of the adenoma.

78. Pituitary adenomas are best classified according to functional hormone


output. This information may be derived from which of the following?
a. Hematoxylin and eosin staining
b. Immunohistochemical staining of pituitary tissue
c. In situ hybridization studies
d. Selective venous sampling from the inferior petrosal sinuses
Answer: b, c, d
Pituitary adenomas have been classified historically as acidophilic, basophilic
and chromophobic. Adenomas may show a variable staining pattern with
conventional hematoxylin and eosin dyes, so it is difficult to classify
adenomas based on these stains. Immunohistochemistry, ultrastructural
studies and in situ hybridization analyses for specific hormones are the most
reliable methods of classifying pituitary adenomas today.

Immunohistochemical staining of pituitary adenomas with specific antibodies


has reliably classified adenomas using highly purified polyclonal and
monoclonal antibodies against prolactin, GH, ACTH, FSH-b LH-b, and TSHb. Many studies with these antibodies have revealed that some pituitary
tumors are composed of several cell types, which produce various hormones.
Some adenomas may not store specific hormones, so immunohistochemical
staining may be weak or absent. The mRNA is usually present in the
cytoplasm of adenomas. Localization of mRNA for specific protein hormones
is becoming more widely used in the classification of pituitary adenomas.
Null-cell adenomas constitute up to 25% of pituitary neoplasms. Selective
venous sampling from the inferior petrosal sinus via transfemoral
catherization is an effective method to compare venous effluent from the
pituitary to systemic levels for a specific hormone. In addition, this technique
may demonstrate laterality. This latter issue is potentially important as certain
small adenomas may not be discernible from the gross appearance at surgery.

79. Which of the following statements is true with respect to Cushing


Disease?
a. Pituitary microadenomas are often small and deep within the gland itself
b. The treatment of choice for hypercortisolism due to a pituitary adenoma in
women of childbearing age is transsphenoidal total hypophysectomy
c. Patients who fail to remit with both surgery and radiation to the pituitary
require either medical or surgical adrenalectomy
d. The long-term recurrence rate after resection of an ACTH-producing
pituitary microadenoma is approximately 40%
Answer: a, c
Pituitary microadenomas secreting ACTH may be very small and are often

located deep within the gland itself. If the tumor is not evident on opening the
dura, incisions must be made into the gland and internal exploration carried
out. If no tumor is identified, then a decision must be made as to whether to
resect all or a portion of the gland. If the endocrine evidence is convincing for
pituitary origin and the patient has no desire to have children, then total
hypophysectomy is warranted. If the petrosal sinus sampling clearly indicates
laterality then appropriate hemiresection of the gland may be done.
About 75% of patients have microadenomas as the source of ACTH
secretion. The postoperative remission rate in these patients is 88% to 96%,
and the long-term recurrence rate appears to be no more than 5%. 10% to
20% of patients who undergo exploration have macroadenomas, and the
postoperative remission rates in these patients have been reported to be from
33% to 61%. Most of these patients require postoperative radiation therapy.
Patients who fail both surgery and radiation require either surgical
adrenalectomy or medical suppression of adrenal function.

80. A 45 year-old woman presents for evaluation of hypertension, recent


onset obesity, hirsutism and depression. Cerebral MRI does not show a
pituitary lesion. Evaluation may include determination of which of the
following?
a. AM serum cortisol levels after low dose dexamethasone suppression
b. Simultaneous serum ACTH measurement in peripheral and inferior
petrosal sinus sites
c. Chest and abdominal CT scan
d. Urinary free cortisol excretion
Answer: b, c, d
The findings of Cushing Syndrome often include central obesity,

hypertension, hirsutism, fatigue, easy bruisability, stria, moon-like facies,


dorsal fat pad, and often depression or other mental changes. Less common
abnormalities include headache, osteoporosis, diabetes mellitus, galactorrhea,
peripheral edema and amenorrhea. Often, a patient presents without the
classic cushingoid appearance and complains only of severe fatigue or
depression. The cause of hypercortisolism is an ACTH-secreting pituitary
adenoma (Cushing disease) in up to 80% of cases, with remainder due either
to an adrenocortical tumor or to an ectopic neoplasm secreting ACTH or
corticotropin-releasing factor. Pituitary-dependent hypercortisolism is much
more common in women(80%) and an ectopic etiology more common in
men.
Up to 60% of patients with pituitary etiologies have nondiagnostic imaging
studies, therefore, the diagnosis often relies completely on endocrine testing.
Multiple measurements of cortisol and ACTH to evaluate the diurnal pattern
are important but often misleading. They are mainly of value when clearly
elevated. The determination of urinary free cortisol excretion over 24 hours is
an extremely important measurement. If the overnight dexamethasone
screening test yields an 8 AM serum cortisol level of less than 5 ug/dl, then
hypercortisolism is rarely present. Generally, patients with a pituitary
etiology of hypercortisolism do not show suppression with the low-dose
dexamethasone test, but do with the higher dose test. Patients with adrenal or
ectopic etiologies do not experience suppression with either dose. Chest and
abdominal CT scans are appropriate to look for adrenal or lung tumors. The
most specific test when the MRI is negative and evidence implicates the
pituitary, is simultaneous measurement of ACTH levels in both inferior
petrosal sinuses and a concurrent determination of the peripheral ACTH
level. This approach produces specific information about the existence of an
ACTH-secreting pituitary tumor and even the laterality of the tumor.

81. The most common mass lesion in the sella turcica is which of the
following?
a. Craniopharyngioma
b. Aneurysm
c. Benign pituitary cyst
d. Pituitary adenoma
Answer: d
Pituitary adenomas are the most common mass lesions in the sella turcica or
parasellar region. They constitute 8% to 10% of all brain tumors.
Occasionally, they are cystic and may be confused with other lesions.
Craniopharyngiomas are the next most common tumor, although these are
more often suprasellar in location. These are more common in children, but
up to one third occur in adults. They are usually cystic and are calcified in
70% of children and 40% of adults. More rare lesions include meningiomas,
germinomas, metastatic malignancies from lung and breast primaries,
gliomas, dermoids and benign epidermoid. Rathke cysts, aneurysms, and a
variety of inflammatory and granulomatous processes.

82. Pharmacologic treatment of growth hormone (GH) excess secondary to a


pituitary adenoma may include the use of which of the following?
a. Bromocriptine
b. Vasopressin
c. Octeotide
d. Prednisone
Answer: a, c

Bromocriptine, a dopamine receptor agonist, has been demonstrated to lower


GH levels in 71% of 126 acromegalic patients. A clinical response was
achieved in up to 95% of acromegalic patients, and reduced somatomedin C
levels were found in some patients with persistently elevated GH levels.
Bromocriptine does not appear to be an effective primary treatment for
acromegaly, but may help to control GH and somatomedin C levels as an
adjuvant therapy. A somatostatin analogue, octeotide, has recently been used
to treat acromegaly and has been demonstrated to significantly reduce GH
and somatomedin C levels in most patients and normalize values in 50%.
This treatment provides only minimal tumor shrinkage, and GH levels rise
again immediately following cessation of the drug. This drug may prove to be
useful as a preoperative treatment or in surgical failures. Vasopressin and
prednisone have no role in the treatment of acromegaly.

You might also like